Lp3 Physics For Engineers

You might also like

Download as pdf or txt
Download as pdf or txt
You are on page 1of 72

BSCE, BSEE, BSECE, AND BSCpE

Phys 1 (Physics for Engineers)


LEARNING PACKET 3

Engr. Maricris Ediza

Engr. Rhea Mae Biñar

Engr. Homer Gaganao

College of Engineering
3 | Physics for Engineers 112

UNIT 9: OSCILLATION AND WAVES

9.0. Intended Learning Outcomes


By the end of this unit, you should be able to:
a. Define wave and its medium
b. Identify the different types of waves.
c. Differentiate pulse wave and periodic waves.
d. Discuss the different characteristics of waves.
e. Relate wave velocity, wavelength, and wave frequency.
f. Discuss the laws in a vibrating string.
g. Describe how waves behave in an obstacle, pass into another medium,
or pass through another wave.
h. Explain what happens when two waves interfere.
i. Distinguish between constructive and destructive interference.
j. Explain how standing waves are formed.
k. Solve simple harmonic motion applications.
l. Define simple harmonic motion (SHM)
m. Define period, amplitude, and frequency.
n. Calculate the restoring force using Hooke’s Law.
o. Describe how force, velocity and acceleration change as an object
vibrates with SHM.
p. Identify the factors that affects the period of a pendulum.
q. Calculate the period and frequency of an object in SHM.

9.1. Introduction
Although we cannot see it in our naked eye, waves come to us in our
surroundings in various forms. Waves carries energy (not matter) from one
place to another. The sun reaches us as light waves, the sound of people
speaking travels to our ears as sound waves, radio, and television connection
travel to our homes in a form of electromagnetic wave.

Many kinds of waves do need a medium through which it can travel. A


medium is any substance where a wave can travel. In short, it is the
surrounding of the wave which allows it to travel. Some waves however do not
need a medium to travel, these are called electromagnetic waves which include
light waves, x-ray waves and radio waves.

Waves exhibit different properties depending on many factors in its


surrounding and medium. Some waves move in straight lines like the wave in
the ocean, some spread out in circles like ripples on a pond and some spread
out in spheres like sound waves in air. Moreover, when a wave encounters or
3 | Physics for Engineers 113

interacts with a boundary it exhibits properties such as reflection, refraction,


diffraction, and interference.

Oftentimes, when we observe a motion of a particular object, it involves a single


action and eventually stop over a period, like a coin falling in a deep well. This
motion will possibly take place once, and after which the coin will remain at
rest. In the case of waves however it is different. Waves exhibits a periodic
motion or oscillation meaning the motion involves a repeated action that
occurs back and forth over a same path in a regular cycle. This kind of motion
is observed as a wrecking ball swaying to and from, a body on a spring (mass-
spring system), the motion of a pendulum and many more.

9.2. Discussions and Assessments


9.2.1. Wave: The Nature of Waves
Figure 9.1 shows the different types of waves and its definitions. Let us focus
on the mechanical wave, a simple way to demonstrate mechanical wave motion
is to flip one end of a rope whose opposite end is attached to a fixed point. Upon
flipping, a wave that consist of a single travelling disturbance is called a pulse
wave; if you continue to generate pulses at one end of the rope, these pulses are
called periodic waves.

Waves
A wave is a vibratory disturbance that
moves through a medium.

Mechanical Waves Electromagnetic Waves


A wave that needs a medium to A wave that does not need a
travel. Examples are water waves, medium to travel. Examples
sound waves, and waves that are light waves, microwaves
travel along a rope or a spring and radio waves

Transverse Waves
A wave which particles vibrate in the
direction perpendicular to the path
along waves travels
Longitudinal Waves
A wave which particles vibrate in the
direction parallel to the direction of
the wave motion

Surface Waves
combination of longitudinal and
transverse wave

Figure 9.1: Classification of Waves


3 | Physics for Engineers 114

A good example of a transverse wave is when one end of a rope is fastened to


a wall and then twitch into an up and down motion, the crest is the maximum
upward displacement while the trough is the maximum downward
displacement. For longitudinal wave, a perfect example is when you move a
spring into a push and pull movement wherein you may observe coils that are
closer together (compression) and some are further apart (rarefaction).

compression rarefaction

Figure 9.2: Transverse & Longitudinal Wave


Source: https: byjus.com/physics/types-of-waves

Characteristics of Waves

All waves have several common characteristics which are as follows:


▪ Amplitude (Symbol: A)
The amplitude of a wave is the maximum displacement of any particle in
medium relative to its rest point. The energy content of a mechanical wave
is characterized by its amplitude. Greater amplitude can transfer more
energy and do more work. The amplitude of the wave does not affect its
wavelength, frequency or velocity.

▪ Period (Symbol: T)
The period of a wave is the time taken to complete one cycle.

▪ Frequency (Symbol: f)
The frequency is the number of waves executed by a particle each second,
it is the reciprocal of period.
1
𝑓 = 𝑇 (cycles/s)
3 | Physics for Engineers 115

Note: unit for frequency is Hertz (Hz) and as in case of oscillation 1Hz =1
cycles/s

Some common multiple of Hz used for high frequency:

1 kilohertz = 1 kHz = 1 x 103 Hz


1 megahertz = 1 MHz = 1 x 106 Hz
1 gigahertz = 1 GHz = 1 x 109 Hz

▪ Wavelength (Symbol: 𝝀)
The wavelength is the distance along the direction of the propagation
between corresponding points on a wave.

▪ Velocity (Symbol: v)
The velocity of a wave is the distance which each wave moves per second.

Relationship between the wave velocity, wavelength, period, and


frequency is describe by:

𝜆
𝑣= = 𝜆𝑓
𝑇

Points along the waves that have the same displacement and are
moving in the same direction at the same time are in phase while points
that have the opposite displacements and are moving in the opposite
direction are said to be out of phase.

In Figure 9.3,” Points C” are in phase while “Point C & Point T” are
out of phase.

Figure 9.3: Characteristics of Wave


3 | Physics for Engineers 116

Laws of Vibrating String

The following laws are behavior of a vibrating string in which the


relationship of frequency (f), tension (T), diameter (d), density (D), and
length (L) are discussed:
1. The Law of Length
The frequency of a vibrating string is inversely proportional to its
length if other factors is considered the same.

𝑓1 𝐿2
=
𝑓2 𝐿1

2. The Law of Tension


The frequency of a vibrating string is directly proportional to the
square root of the tension if other factors is considered the same.

𝑓1 𝑇1
=√
𝑓2 𝑇2

3. The Law of Diameter


The frequency of a vibrating string is directly proportional to its
diameter if other factors is considered the same.

𝑓1 𝑑2
=
𝑓2 𝑑1

4. The Law of Density


The frequency of a vibrating string is inversely proportional to the
square root of its density if other factors is considered the same.
𝑓1 𝐷2
=√
𝑓2 𝐷1

Sample Problems_____________________________________________________

1. A sound wave has a frequency of 250 hertz. What is the period of the sound
wave?

Given: f = 250 Hz
Required: T
1
Solution: 𝑇 = 𝑓
1
𝑇 = 250 𝐻𝑧
𝑇 = 0.004 𝑠
3 | Physics for Engineers 117

2. Transverse waves travelling along a rope have a frequency of 12.40 Hz and


are 3 m long. What is the velocity of the waves?

Given: f = 12.40 Hz
𝜆=3m
Required: v
Solution: 𝑣 = 𝑓𝜆
𝑣 = (12.40 𝐻𝑧)(3𝑚)
𝑣 = 37.2 𝑚/𝑠

3. Water waves in a small tank arc 6 cm long. They pass a given point at the
rate of 4.8 waves per second.
(a.) What is the speed of the waves?
(b.) What is the period of these waves?

Given: 𝜆 = 6 cm
f = 4.8 vib./s = 4.8 Hz
Required: (a.) v
(b.) T

Solution: (a.) 𝑣 = 𝑓𝜆 = (4.8 Hz)(6 cm) = 28.8 cm/s


1 1
(b.) 𝑇 = 𝑓 = 4.8 𝐻𝑧 = 0.21𝑠

9.2.2. Wave: Properties and Interaction

Reflection

When a wave hits a surface through which it cannot pass, it bounces back. This
phenomenon is called reflection. Examples of such property of wave are
reflection of light from a mirror or any smooth polished surface, the reflection
of waves from the edge of a pool, and the moon wherein it does not emit any
light of its own, it only reflects the light of the sun.

Reflection may be partial or complete depending on the nature of the reflecting


surface. If there is no mechanism for extracting energy from the wave, all the
energy incident with the wave is reflected.

Reflected wave behave differently depending to its medium. Table 9.1 below
shows how it exhibits distinct behavior when a wave reaches a boundary of
different medium.
3 | Physics for Engineers 118

Table 9.1: Behavior of Wave at Different Boundary

Boundary Wave
More rigid medium Inverted
Less rigid medium Erect
New Medium Some part is transmitted,
and some part is reflected

An idea on what happens when a wave is reflected can be observed when you
bounce a tennis ball on the floor. The larger the angle at which the ball strikes
the floor, the larger the angle from where it bounces off the floor.

A ray diagram is used to show the direction of the incident and reflected waves.
The ray that hits the barrier is called incident ray while the ray that is thrown
back is called reflected ray. The line perpendicular to the surface is called
normal.

Figure 9.4: A ray diagram showing the direction of


incident and reflected rays of a bouncing ball
Source: Padua, A.L. & Crisostomo, R. M. (2010). Science and Technology IV Practical and
Explorational Physics Modular Approach, 2nd Edition

The angle formed by the incident ray with the normal is called angle of
incidence (∠𝒊) and the angle formed by the reflected ray with the normal is
called angle of reflection (∠𝒓). The relationship of these angles is described in
the Law of Reflection that states that:

1. When a wave is reflected at the boundary of a medium, the angle of


incidence is equal to the angle of reflection.
2. The incident ray, the reflected ray, and the normal lie on a single plane.
3 | Physics for Engineers 119

Refraction

When a wave moves from one medium into another, medium at an angle, it
changes speed as it enters to a second medium, which causes it to bend. This
phenomenon is called refraction. Refraction is the bending of waves due to a
change in speed as it passes from one medium to another medium of different
density.

The principle of refraction gives rise to such familiar effects as the apparent
distortion of objects partly submerged in water. A spoon appears to be broken
when places in a glass of water because the light waves bend when they pass
from air to glass to water.

Water waves travel at different speeds at different depths. They travel slower
in shallow water than in deep water. Consider the Figure 9.5 below, when the
wave is moving from deep to shallow water, the angle of incidence is greater
than the angle of refraction. When the waves are moving from shallow to deep
water, the angle of incidence is smaller than the angle of refraction.

Figure 9.5: Waves moving from deep to shallow water (A); waves move from
shallow to deep water (B)
Source: Padua, A.L. & Crisostomo, R. M. (2010). Science and Technology IV Practical and
Explorational Physics Modular Approach, 2nd Edition
3 | Physics for Engineers 120

In general, if a wave slows down just after passing the boundary


between two media, the angle of refraction is less than the angle of incidence.
If the wave speeds up, the angle of refraction is greater than the angle of
incidence.

Diffraction

The bending of waves around the edge or corner of a barrier is known as


diffraction. The amount of diffraction is dependent on the wavelength and the
size of the opening barrier. Figure 9.6 shows how wave diffract. Waves can go
through a hole in a barrier and spread out and it can also bend around the
barrier.

Diffraction is only significant if the size of the opening is about the size of the
wavelength. Larger openings produce less diffraction.

Figure 9.6: Waves passing through a hole barrier (A) and bending around a barrier (B).
The spreading out of waves in the oceans (C)
Source: Padua, A.L. & Crisostomo, R. M. (2010). Science and Technology IV Practical and
Explorational Physics Modular Approach, 2nd Edition
3 | Physics for Engineers 121

Interference

When two waves meet, they influence each other. This interaction is called
interference. The interference waves follow the Superposition Principle which
states that, when two waves meet while travelling through a medium at the same time,
the result is a wave whose displacement is equal to the vector sum of the displacement
of two waves.

Interference of incoming radio waves with those reflected from the clouds or
airplanes can sometimes be observed on your television or radio set. Similarly,
the colorful swirling rainbow effect on your soap bubbles and in oil films
floating in the streets on a rainy day are produced by interference waves.

The nature of interference depends on the relative positions of their crest and
troughs. There are two types of interference: constructive and destructive.

If the crest of one wave meets the crest of the other, and the troughs of one wave
meet the troughs of the other, the result is called constructive interference. In
this case the two waves are in phase. The energy from the two waves will
combine. Thus, the amplitude of the resulting wave will be twice the amplitude
of original waves.

On the other hand, if the crest of one wave meets the troughs of the other,
destructive interference occurs. In this case, the two waves are out of phase.
The amplitude of the first wave cancels out the amplitude of the second wave.
This type of interference produces a wave with zero amplitude.

Figure 9.7: In constructive interference, the waves pass through and reinforce each
other (A).
In destructive interference, the waves cancel out each other (B).
Source: Padua, A.L. & Crisostomo, R. M. (2010). Science and Technology IV Practical and
Explorational Physics Modular Approach, 2nd Edition
3 | Physics for Engineers 122

Standing Waves and Natural Frequency

When two waves of the same frequency, wavelength, and amplitude travel in
opposite directions and interfere, a new pattern will be established due to the
superposition of the reflected waves with the original one. This pattern is called
stationary waves or standing waves.

Standing waves are generated when you pluck a guitar string, bow a violin, or
strike a piano string. These waves are also produced in vibrating air columns
in wind instruments and organ pipes, and in the throat, mouth and nose of a
person speaking.

The points on a standing wave vibrate with varying amplitudes. The point on
the standing wave that always undergoes complete destructive interference
and with zero amplitude is called the node. The point halfway between two
nodes which at maximum amplitude occurs is called antinode.

Figure 9.8: Nodes and Antinodes of a Standing Wave


Source: Padua, A.L. & Crisostomo, R. M. (2010). Science and Technology IV Practical and
Explorational Physics Modular Approach, 2nd Edition
3 | Physics for Engineers 123

Figure 9.9: Resonant Frequencies


Source: Padua, A.L. & Crisostomo, R. M. (2010). Science and Technology IV Practical and
Explorational Physics Modular Approach, 2nd Edition

Only certain frequencies produce standing wave patterns. The frequencies at


which large amplitude standing waves are produced are called natural
frequencies or resonant frequencies.

A stretched string can be analyzed to determine its natural frequencies. The


number of closed segments or loops of standing waves that will fit between
nodes at the end is equal to an integral number of half-wavelengths.

Notice in Figure 9.9 – B that the nodes are at the end of the string and the
antinode is at the middle. This gives the wave the standing still appearance,
hence, the name standing waves. Doubling the frequency of vibration produces
one more node and one more antinode in the string as in Figure 9.9 – C. As the
frequency of vibration is increased, the number of nodes and antinodes
increases.

The natural frequency of oscillation, where 𝑣 is the wave speed and 𝐿 is the
length of the string is:
𝑣 𝑛𝑣
𝑓𝑛 = 𝜆 = , for n = 1, 2, 3, …
𝑛 2𝐿

Thus,
3 | Physics for Engineers 124

𝑣𝜆𝑛
𝐿= , for n = 1, 2, 3, …
2

𝑣
The lowest natural frequency (𝑓1 = 2𝐿) is called fundamental
frequency. All other natural frequencies are integral multiples of the
fundamental frequency: 𝑓𝑛 = 𝑛𝑓1, (n = 1, 2, 3, …). The set of frequencies 𝑓1, 𝑓2 =
2𝑓1 , 𝑓3 = 3𝑓1 and so on, is called a harmonic series, 𝑓1 being the first harmonic,
𝑓2 as second harmonic, and so on.

9.2.3. Oscillation: The Simple Harmonic Motion

A periodic motion or oscillation occurs because of a restoring force.


Whenever a body is pulled away from its equilibrium position (position at rest),
the net force on the system pulls it back towards its equilibrium position. This
net force is called restoring force because it attempts to restore the body
towards its initial equilibrium position.

The simplest oscillation is Simple Harmonic Motion (SHM). A motion is


said to be in simple harmonic motion (SHM) if the restoring force is directly
proportional to the displacement of the object in motion, 𝐹𝑅 ∝ ∆𝑥. Thus, its
acceleration is also proportional to the body’s displacement, 𝑎 ∝ ∆𝑥.

To understand more about simple harmonic motion, cycle, amplitude,


period, and frequency should be considered. A cycle is a complete round trip
motion of a body. That is, when a body moves from its starting point and travels
back to its initial position, the body is said to complete one cycle. The amplitude
(designated as A) refers to the maximum displacement of a body from its initial
equilibrium position. The period (designated as T) is the time needed to
𝑡𝑖𝑚𝑒
complete one cycle, in equation form: 𝑇 = 𝑐𝑦𝑐𝑙𝑒. The frequency (designated as f)
is the opposite of period, which is the number of cycles per time, in equation
𝑐𝑦𝑐𝑙𝑒𝑠 1
form: 𝑓 = = 𝑇. It can also be expressed as cycle per second or hertz (Hz).
𝑡𝑖𝑚𝑒

Let us consider a body that undergoes simple harmonic motion: a body of a


specific mass attached to a spring and a simple pendulum.
3 | Physics for Engineers 125

Mass – Spring System

Equilibrium Consider a mass-spring


Position system as shown in Figure 1. If an
applied force is subjected into the
Position 1 system (spring is pulled from its
equilibrium position), assuming
there is no air resistance
Position 2
(frictionless), the system will
behave in such a way as to restore it
form its initial equilibrium position
and it will continue to move
Position
3 periodically unless another force is
-A 0 A applied to stop the movement.
X3 X2 X1
Once pulled, the motion in the
Figure 1: Mass – Spring System figure will move from position 1, 2,
3, 2, 1, 2, 3, 2,1… and so on. An
example for a complete cycle for this
motion is from position 1, 2, 3, 2 and back to position 1. The amplitude will be
the maximum displacement from position at rest, that is, from position 2 to
position 1 (positive amplitude) or from position 2 to position 3 (negative
amplitude). The period will be the time it takes for the motion to go back from
its initial position; hence, it will be the time it takes from position 1 and back.
Accordingly, the frequency will be the reciprocal of the period.

Hooke’s Law
Periodic motion like SHM displayed in Figure 1 is caused by a restoring
force. The magnitude of this force can be calculated using equation proposed
in Hooke’s Law,

𝐹𝑅 = −𝑘∆𝑥 Equation 1

Where FR is the restoring force or for a mass – spring system it is sometimes


called spring force, k is the spring constant wherein its magnitude varies
depending on the type of spring, and ∆𝑥 is the displacement of the spring from
equilibrium position. The negative sign in the equation indicates that the
direction of the restoring force is opposite to the direction of the mass’
displacement.
3 | Physics for Engineers 126

Sample Problems____________________________________________________

For a mass-spring system shown in Figure 1,

4. Calculate the spring force if the amplitude is 10 cm and the spring


constant is 200 N/m.

Given: a = ∆𝑥 = 10 cm
k = 200 N/m
Required: FR
Solution:
𝐹𝑅 = −𝑘∆𝑥
200𝑁 1𝑚
𝐹𝑅 = − ( ) (10 𝑐𝑚 ∙ 100 𝑐𝑚)
𝑚
𝐹𝑅 = −20𝑁

5. Calculate the displacement of the spring if the spring force is 1KN and the
spring constant of the spring is 114 N/m.

Given: FR = 1KN = 1000 N


k = 114 N/m
Required: ∆𝑥
Solution:
𝐹𝑅 = −𝑘∆𝑥
114𝑁
1000N = − ( ) ∆𝑥
𝑚
∆𝑥 = −8.77 m

6. Calculate the spring constant for a spring if the amplitude is 15cm and the
restoring force is 35 N.

Given: FR = 35 N
a = ∆𝑥 = 15 cm
Required: k
Solution:
𝐹𝑅 = −𝑘∆𝑥
1𝑚
35N = −(𝑘 ) (15 𝑐𝑚 ∙ )
100𝑐𝑚
𝑘 = −233.3 𝑁/𝑚
3 | Physics for Engineers 127

SHM: Force, Velocity and Acceleration

One might ask what happens to the force, velocity, and acceleration when an
object behave in SHM. To answer this problem, let us again consider the mass – spring
system shown in Figure 1. There are three locations
that is observed in the figure, position 2 is when the
mass-spring system is at rest or at equilibrium.
Positions 1 and 3 are when the mass in the spring is at
maximum displacement from equilibrium position
(position 2).

Now, let us observe position 1 and 3 separately as


shown in Figure 2. The direction of the velocity at these
specific location changes, that means that the velocity
at these positions is zero (𝑣1 = 0 & 𝑣3 = 0). This can also
be observed in free falling bodies, when a ball is
thrown upward, the point where the ball changes direction has zero velocity. So, if the
velocity of the two endpoint is zero, just like in projectile motion, we can safely say
that the magnitude of the velocity at the middle is at maximum ( |𝑣2 | = max).

The force that governs in this system can be explained


according to Hooke’s Law, 𝐹𝑅 = −𝑘∆𝑥.

The displacement at positions 1 and 3 are at maximum


magnitude and from the previous discussion it is
called the amplitude, A, (|𝑥1 | = max = A & |𝑥3 | = max =
A).

To determine the forces at these positions, we use


Hooke’s Law, |𝐹1 | = −𝑘𝑥1 = 𝑚𝑎𝑥 and |𝐹3 | = −𝑘𝑥3 =
𝑚𝑎𝑥, thus, the magnitude of forces at these positions
are at maximum.
The magnitude of the displacement at position 2 is zero (𝑥2 = 0) therefore the
magnitude of the force at this position is also zero |𝐹2 | = −𝑘𝑥2 = 0.
3 | Physics for Engineers 128

Using Newton’s Second Law of Motion we can


determine the magnitude of the acceleration using
the magnitude of the spring force. ∑ 𝐹𝑥 = 𝑚𝑎 and
the only force that acts on this system is the spring
force thus,

∑ 𝐹𝑥 = 𝑚𝑎

𝐹𝑅 = 𝑚𝑎
Based on the equation, the acceleration is
proportional to the spring force so the magnitude
of the acceleration at position 1 and 3 are at
maximum and zero at position 2 (|𝑎1 | = max, |𝑎3 | = max and |𝑎2 | = 0).

To prove that the magnitude of the velocity at


equilibrium is at maximum, let us consider another
position where the mass in the spring is moving away
(position 4) and towards equilibrium (position 5) in
Figure 4. The spring force always tends towards
equilibrium that is why it is also called the restoring
force. The acceleration is proportional to the spring
force therefore the direction of the acceleration also
tends towards equilibrium.

Upon observation on the free-body diagram in Figure


5, position 4 is when the motion of the system is
slowing down while position 5 is when it is speeding up
therefore at equilibrium the magnitude of the velocity is at maximum.

Equations for Simple Harmonic Motion (SHM)

To derive equations for Simple Harmonic Motion (SHM), we need to first recall
what we have learned in the previous lessons involving Uniform Circular Motion
(UCM) since UCM when viewed from the side (2-dimensional projection along x or y
axis) is SHM.
3 | Physics for Engineers 129

Consider Figure 5.a, a ball is moving along the x and y plane in a uniform
circular motion, if this movement is viewed at a different angle (side view) shown in
Figure 5.b the movement of the ball will be an up-down movement from the positive
y to the negative y axis similar to a simple harmonic motion.

SHM: Equations for Position

For UCM:
𝑜𝑝𝑝 𝑦 𝑎𝑑𝑗 𝑥
𝑠𝑖𝑛𝜃 = ℎ𝑦𝑝 = 𝑐𝑜𝑠𝜃 = ℎ𝑦𝑝 =
𝑟 𝑟
𝑦 = (𝑟)𝑠𝑖𝑛𝜃 𝑥 = (𝑟)𝑐𝑜𝑠𝜃

𝜃
Recall: 𝜔 = 𝑡 and 𝜃 = 𝜔𝑡
Note that for UCM when viewed from the side is
SHM. Thus, radius of the circular path is equal to the
amplitude in SHM (r = A)

𝑦 = (𝐴)sin (𝜔𝑡) Equation 2.1


𝑥 = (𝐴)cos (𝜔𝑡) Equation 2.2

𝜃
To convert the time (t) into period (T) in the equation 𝜔 = 𝑡 , we need 𝜃 = 1 cycle =
2𝜋,
2𝜋
𝜔= Equation 2.3
𝑇

2𝜋 1
Rearranging, 𝑇 = and 𝑇 = 𝑓
𝜔
𝑇=𝑇
2𝜋 1
=
𝜔 𝑓
𝜔 = 2𝜋𝑓 Equation 2.4

Substituting 𝜔 = 2𝜋𝑓 to Equation 1.1 and 1.2,


𝑦 = (𝐴)sin (2𝜋𝑓𝑡) Equation 2.5

𝑥 = (𝐴)cos (2𝜋𝑓𝑡) Equation 2.6


3 | Physics for Engineers 130

SHM: Equations for Velocity

To determine equations for velocity we use equation for position,

𝑥 = (𝐴)cos (𝜔𝑡) Equation 2.2


𝑑𝑥
Note that 𝑣 = 𝑑𝑡
𝑑𝑥
= −(𝐴)[sin (𝜔𝑡)](𝜔) Equation 3.1
𝑑𝑡
Thus,

𝑣 = −(𝐴)(𝜔)[sin (𝜔𝑡)] Equation 3.2

SHM: Equations for Acceleration


𝑑𝑣
To determine equations for acceleration we can use equation for velocity since 𝑎 = ,
𝑑𝑡

𝑣 = −(𝐴)(𝜔)[sin (𝜔𝑡)] Equation 3.2


𝑑𝑣
= −(𝐴)(𝜔)[cos (𝜔𝑡)](𝜔)
𝑑𝑡
Simplifying,
𝑑𝑣
= −(𝐴)(𝜔2 )[cos (𝜔𝑡)]
𝑑𝑡

𝑑𝑣 𝑑𝑥 𝑑2 𝑥
Since 𝑎 = and 𝑣 = then, 𝑎 =
𝑑𝑡 𝑑𝑡 𝑑𝑡 2

𝑑𝑣 𝑑2 𝑥
𝑎= = = −(𝐴)(𝜔2 )[cos (𝜔𝑡)] Equation 4.1
𝑑𝑡 𝑑𝑡 2

This can be simplified further since 𝑥 = (𝐴)cos (𝜔𝑡) from Equation 2.2,

𝑎 = −(𝜔2 )𝑥 Equation 4.2 (Condition for SHM)

Equation for Mass – Spring System

For a mass-spring system in Figure 1,

∑ 𝐹𝑥 = 𝑚𝑎

−𝐹𝑅 = 𝑚𝑎
Using Hooke’s Law, 𝐹𝑅 = −𝑘∆𝑥 in Equation 1,

−𝑘𝑥 = 𝑚𝑎
𝑘
𝑎 = −𝑚𝑥 Equation 4.3
3 | Physics for Engineers 131

Substituting Equation 4.3 to Equation 4.2,

𝑎 = −(𝜔2 )𝑥 Equation 4.2 (Condition for


SHM)
𝑘
− 𝑚 𝑥 = −(𝜔2 )𝑥
Simplifying,
𝑘
𝜔2 = 𝑚 Equation 4.4
𝑘
𝜔 = √𝑚 Equation 4.5

To determine the period for a mass-spring system we use Equation 4.5 and substitute
2𝜋
it to 𝜔 = in Equation 2.3, and simplify we get,
𝑇
𝑚
𝑇 = 2𝜋 √ 𝑘 Equation 4.6
And frequency is the reciprocal of period, thus,

1 1 𝑘
𝑓 = 𝑇 = 2𝜋 √𝑚 Equation 4.7

Simple Pendulum

If the pendulum’s the angle of displacement is relatively small (approximately


less than 150 ) and operating at a frictionless environment, the pendulum is assumed
to exhibit a simple harmonic motion (SHM). Consider Figure 7 below, position 2 is
when the spring is at equilibrium. When an applied
force is subjected, that is, pulling the body’s center
of gravity away from equilibrium position to the left
or right (position 1 or position 3), the system will
behave in such a way as to restore the body from its
initial equilibrium position.

Notice that when the body is at maximum


displacement (position 1 and 3) from equilibrium,
the magnitude of the force (restoring force) will be
at maximum and gradually decreases as it
approaches position 2 wherein the magnitude of the
force is zero.

To derive equations for simple pendulum, consider


the free-body diagram in Figure 8, the force that acts
in the system is the restoring force,

𝐹𝑅 = −𝑚𝑔𝑠𝑖𝑛𝜃 Equation 5.1


3 | Physics for Engineers 132

For the pendulum to perform SHM, Ɵ should be relatively small, thus, we can safely
assume that 𝜃 ≈ 𝑠𝑖𝑛𝜃

𝐹𝑅 = −𝑚𝑔𝜃 Equation 5.2

Recall that angular displacement (Ɵ) is equal to the


arclength (s) divided by radius (r) which is equal to
the length (L) of the string in the pendulum in the
free-body diagram,
𝑠
𝜃=𝐿 Equation 5.3

𝑠 = 𝜃𝐿 Equation 5.4

Substituting Equation 5.3 to Equation 5.2,


𝑠
𝐹𝑅 = −𝑚𝑔 𝐿 Equation 5.5

By Hooke’s Law,

𝐹𝑅 = −𝑘∆𝑥 Equation 1

Note that for a pendulum the arclength is ∆𝑥,

𝐹𝑅 = −𝑘𝑠 Equation 5.6

Substituting Equation 5.5 to Equation 5.6 and simplifying,


𝑠
−𝑚𝑔 𝐿 = −𝑘𝑠
𝑚𝑔
𝑘= Equation 5.7
𝐿

To determine the period of the pendulum, substitute Equation 5.7 to Equation 4.6,

𝑚
𝑇 = 2𝜋 √ 𝑘 Equation 4.6
𝑚
𝑇 = 2𝜋 √ 𝑚𝑔
𝐿

𝐿
𝑇 = 2𝜋 √𝑔 Equation 5.8
Consequently, frequency is the reciprocal of period, thus,

1 1 𝑔
𝑓 = 𝑇 = 2𝜋 √ 𝐿 Equation 5.9
3 | Physics for Engineers 133

Sample Problems___________________________________________________________

7. What will be the frequency of a simple pendulum if the length of the string is
5cm?

Given: L = 5 cm
Required: f
1 𝑔
Solution: 𝑓 = 2𝜋 √ 𝐿

1 9.8 𝑚/𝑠 2
𝑓 = 2𝜋 √ 1𝑚
5𝑐𝑚 ∙
100 𝑐𝑚

2.23
𝑓= 𝑜𝑟 2.23 𝐻𝑒𝑟𝑡𝑧 (𝐻𝑧)
𝑠

8. Determine the period of the pendulum in Sample Problem 1.

Given: f = 2.23 Hz
Required: T
1
Solution: 𝑇=
𝑓
1
𝑇 = 2.23𝐻𝑧
𝑇 = 0.448 𝑠

9. Determine the spring constant of a spring attached with a 12 kg mass that


operates a period of 57 seconds.

Given: m = 12kg
T = 57s
Required: k
𝑚
Solution: 𝑇 = 2𝜋 √ 𝑘

12𝑘𝑔
57𝑠 = 2𝜋 √
𝑘
𝑘 = 0.1458 𝑁/𝑚

10. Determine the frequency of the mass-spring system in Sample Problem 3.

Given: T = 57s
Required: f
1
Solution: 𝑓=𝑇
1
𝑓 = 57𝑠
𝑓 = 0.0175 𝐻𝑧
3 | Physics for Engineers 134

Assessment
Answer the two assessments below (Exercise and Quiz). Write/Encode the
assessments in a long bond paper.

You will be graded based on the following rubrics:


RUBRIC

A. Essay/Quiz

Criteria 5 4 3 2 1
The student was The student
able to elicit ideas The student is unable to
and correct was not only elicit ideas
The student
able to elicit and concepts
understanding was able to The student
ideas and from the
from the reading, elicit ideas and was able to
Understanding correct reading
shows evidence of correct elicit ideas
40% understanding indicating
internalizing these, understanding from the
but also shows that he/she
and contributed from the reading
evidence of has not read
additional thoughts reading.
internalizing the
to the core idea. these. prescribed
reading.
Shows correct Shows
Shows correct
content with Shows irrelevant and
content with
Content additional content Shows correct partial erroneous
evidence
40% resources not content. erroneous content. Did
sourced from
limited to the content. not follow
the module.
module. directions.
Submitted Submitted
Submitted
Submitted on or Submitted after after after
Timeliness after the
before the the deadline: deadline: deadline:
20% deadline:
deadline. 1 day – 3 days 9 days – 15 16 days - 30
4 days – 8 days
days days.
Note for timeliness: 0 points for late submission more than 30 days from the deadline.

Exercise No. 9
1. Violet light has a shorter wavelength than red light. Which has greater frequency? Explain.
2. If the frequency of a vibrating object is doubled, what happens to its period?
3. What is the amplitude and what does it measure?
4. What property of light is evident in the following examples?
a. When catching fish, spearfishermen aim below the image of the fish.
b. A pencil seems to go crooked when dipped in a glass of water.
c. Buildings have glass curtain walls tinted with metallic coatings to keep out the sun’s heat.
d. A laser beam scatters after passing to a small slit.
5. Suppose you dip your finger into a basin of water to make circular waves. What happens to the
wavelength of the wave as frequency increases?
3 | Physics for Engineers 135

6. A pulse reaches a boundary of a medium more rigid than the one from which it came. Is the
pulse erect or inverted?
7. Is it possible for one wave to cancel another wave so that no amplitude remains?
8. What causes standing wave?
9. What happens to the waves when there is:
a. Constructive interference
b. Destructive interference
10. When do waves become:
a. In phase
b. Out of phase
11. How can you tell if an object follows SHM?
12. Does SHM follow a uniformly accelerated motion? Explain your answer.
13. In SHM, what happens to the acceleration when the velocity is at maximum?
14. In SHM, what happens to the velocity when the acceleration is at maximum?

Quiz No. 9

1. If the middle C is stuck on a keyboard,


a. What is the period of one vibration of this tone, if it has a frequency of 256 Hz?
b. As the sound of this middle C tone leaves the instrument at a speed of 340 m/s, what
is its wavelength in air?
2. A mosquito flaps its wings its wings 600 vibrations per second, which produces 600 Hz buzz.
How far does the sound (of the buzz) travel between wings beats if the speed of the sound is
340 meter per second?
3. The wavelength of the signal from Channel 10 is 3.42 meters. Does Channel 10 broadcast on a
frequency above or below the FM radio band which is 88 to 108 MHz?
4. What is the wavelength of the radio waves received at 100 MHz on your radio dial if the radio
waves speed is the same as the light?
5. A sonar signal with frequency of 1.0 x 106 Hertz has a wavelength 1.50 mm in water.
a. What is the speed of the signal in waste?
b. What is its period in water?
6. The AM radio signals are broadcast at frequencies between 550 KHz and 1600 KHz and travel
at 3 x 106 m/s.
a. What is the range of wavelength of these signals?
b. FM frequencies range 88 MHz and 108 MHz and travel at the same speed. What is
the range of FM wavelengths?
7. A radio wave ha frequency of 3 x 107 Hz. It has a wavelength of 10 m long. What is the speed
of the radio wave?
8. Calculate the frequency of a mass-spring system operating at a period of 3 min.
9. What is the mass of mass-spring system shown in Figure 1 if the spring constant is 31 N/m
operating at a period of 5 min.?
10. Calculate the frequency of the mass-spring system in Item No. 9.
11. A simple pendulum has a length of 9 cm. Calculate its frequency.
12. Calculate the period of a pendulum with a length of 2 m.
13. What is the length of the string in a pendulum that operates at 17 Hz?
3 | Physics for Engineers 136

9.3. References
Padua, A.L. & Crisostomo, R. M. (2010). Science and Technology IV Practical and
Explorational Physics Modular Approach, 2nd Edition, Philippines: Vibal
Publishing House Inc.

Young, H.D. (1992). University Physics, 8th Edition, Addison-Wesley Pub. Co.
Review of Simple Harmonic Motion. Flipping Physics. March 23, 2020.
https://www.flippingphysics.com/ap1-shm-review.html.
Collection: OpenStax University Physics - All Volumes Edited by: Cengage
WebAssign URL: https://legacy.cnx.org/content/col12105/1.1/ Copyright:
Cengage WebAssign License: http://creativecommons.org/licenses/by/4.0/
Based on: University Physics <http://legacy.cnx.org/content/col11994/1.1>
arranged by OpenStax University Physics.

Quiseo, J.M. (2012). General Engineering & Applied Sciences Source Book 2nd
Edition. Philippines: GioBooks

https: byjus.com/physics/types-of-waves

Del Rosario, A.C.M. (2004). College Physics.Philippines

Review of Simple Harmonic Motion. Flipping Physics. March 23, 2020.


https://www.flippingphysics.com/ap1-shm-review.html.

9.4. Acknowledgement

The images, tables, figures, and information contained in this module were
taken from the references cited above.
3 | Physics for Engineers 137

UNIT 10: FLUIDS

10.0. Intended Learning Outcomes


By the end of this unit, you should be able to:
a. Distinguish fluid statics from fluid dynamics.
b. Discuss the different properties of fluids - density, pressure, buoyancy,
and surface tension.
c. Determine how pressure varies with depth and density.
d. Calculate the pressure exerted by a fluid.
e. Explain why some objects float while others sink.
f. State Pascal’s Principle and recognize its application.
g. Describe how cohesive and adhesive forces cause surface tension and
capillary action.
h. Cite practical applications of Archimedes’ and Bernoulli’s Principle

10.1. Introduction
A substance that can easily change its shape and is able to flow is called fluid.
Both liquid and gasses are considered fluids. Fluids are generally divided into
two types, namely, ideal fluids and real fluids.

Table 10.1: Ideal and Real Fluids


Ideal Fluids Real Fluids
No viscosity Viscous
(Resistance to shear or
flow is zero)
Incompressible Compressible
Have uniform velocity Non-uniform velocity
distribution when distribution
flowing
No friction between Experience friction
moving layers of between moving layers
fluids of fluids
No eddy currents With eddy currents
No turbulence Turbulent in flow
(Laminar flow)

Real fluids are further divided into two namely:


1. Newtonian Fluids – exhibit constant or uniform viscosities.
2. Non-Newtonian Fluids – viscosities will vary with velocity.
3 | Physics for Engineers 138

Some properties of fluids allow airplanes to fly, boats to float,


submarines to sink and insects to walk on water surfaces. These properties are
density, pressure, buoyancy, and surface tension. The study of fluids is
divided into two categories, it can either be at rest (fluid statics) or in motion
(fluid dynamics). Both will be discussed in this lesson.

10.2. Discussions and Assessments


10.2.1. Density
Density is the ratio of mass per unit volume. Unlike mass and volume, density
is independent of the amount of matter present in a specific substance. This
means that if you break a piece of ruler or any material, each piece may have
half the amount of mass and volume, but its density remains the same before
and after you break it. This type of property is commonly known as intensive
property.

The SI unit for density is kilogram per cubic meter (kg/m3). The symbol used to
represent density is the Greek letter 𝜌 (rho) and is defined by the equation:

𝒎 (𝑴𝒂𝒔𝒔)
𝝆(𝒅𝒆𝒏𝒔𝒊𝒕𝒚) =
𝑽 (𝑽𝒐𝒍𝒖𝒎𝒆)

Density of any liquid:


𝝆𝒅𝒆𝒏𝒔𝒊𝒕𝒚 = 𝝆𝒘𝒂𝒕𝒆𝒓 (𝑺𝑮)𝒍𝒊𝒒𝒖𝒊𝒅
Where:
𝝆𝒘𝒂𝒕𝒆𝒓 = 1 g/cm3
= 1 g/ml
= 1000 kg/m3
= 9810 N/m3
= 62.4 lbm/ft3

Specific Gravity (SG)


- Ratio of the liquid’s density to that of water at 40C.
- It is also called relative density.
𝝆𝒍𝒊𝒒𝒖𝒊𝒅
𝑺𝑮𝒍𝒊𝒒𝒖𝒊𝒅 =
𝝆𝒘𝒂𝒕𝒆𝒓
Specific Volume (v)
- The volume occupied by unit of mass fluid.
- It is the reciprocal of density.
𝟏 𝑽
𝒗 (𝒔𝒑𝒆𝒄𝒊𝒇𝒊𝒄 𝒗𝒐𝒍𝒖𝒎𝒆) = =
𝝆 𝒎
3 | Physics for Engineers 139

Specific Weight (γ, gamma)


- Weight of a fluid per unit volume.
- The SI unit is N/m3
- It is simply the product of the density of the fluid and the acceleration due
to gravity.
𝜸 = 𝝆𝒈

10.2.2. Pressure
Pressure is perpendicular force per unit area that a fluid exerts on any surface
with which it comes in contact. The SI unit is Pascal (Pa) (1 Pa = 1N/m 2).
𝑭 (𝑭𝒐𝒓𝒄𝒆)
𝑷=
𝑨 (𝑨𝒓𝒆𝒂)
The Pressure exerted on an object by a column of fluid is known as
hydrostatic pressure and the pressure exerted by air is referred to as air
pressure or atmospheric pressure (1 x 105 Pa). The instrument used in
measuring the pressure of the atmosphere is called barometer.

Types of Pressure
1. Atmospheric Pressure
The pressure exerted by atmospheric air on any surface. It is measured by
a barometer.

𝑃𝑎𝑡𝑚 = 𝜌𝑔𝐻𝑜

Standard Atmospheric Pressure: (Sea Level)


= 1 atm
= 760 mmHg
= 14.7 psi
= 1.013 bar
= 101325 Pa

2. Gauge Pressure
It is the pressure of a fluid contained in a CLOSED VESSEL. It is ALWAYS
MORE THAN the atmospheric pressure.
𝑃𝑔𝑢𝑎𝑔𝑒 = 𝜌𝑔𝐻
Where: 𝜌 = 𝑑𝑒𝑛𝑠𝑖𝑡𝑦
g = gravitational constant 9.8 𝑚/𝑠 2
H = height/depth
3 | Physics for Engineers 140

3. Vacuum Pressure
It is the pressure of a fluid ALWAYS LESS THAN the atmospheric
pressure.
Ex. Pressure in a steam condenser

4. Absolute Pressure
It is the pressure of a fluid which is measured with respect to ABSOLUTE
ZERO PRESSURE as a reference.

ABSOLUTE ZERO PRESSURE


Molecular momentum is zero (perfect vacuum)

𝑃𝑎𝑏𝑠 = 𝑃𝑎𝑡𝑚 + 𝑃𝑔𝑎𝑢𝑔𝑒


𝑃𝑎𝑏𝑠 = 𝑃𝑎𝑡𝑚 − 𝑃𝑣𝑎𝑐𝑐

To find the pressure exerted by water above an object:

𝑭𝒈 (𝒇𝒐𝒓𝒄𝒆 𝒐𝒇 𝒈𝒓𝒂𝒗𝒊𝒕𝒚)
𝑷=
𝑨 (𝑨𝒓𝒆𝒂)

Force of gravity (Fg) = mg


From the density equation, m = ρV
The volume of the water is V = Ah
Thus,
Fg = mg
Fg = ρVg
Fg = ρAhg

Substituting to the pressure equation,

Fg ρAhg
𝑃= = = 𝜌𝑔ℎ
𝐴 𝐴

Thus, the pressure of water is,

𝑷 = 𝝆𝒈𝒉

Where P is pressure, 𝜌 is density, 𝑔 is gravitational constant and ℎ is the


height.
3 | Physics for Engineers 141

Connection between Hydrostatic Pressure & Atmospheric Pressure

𝑃ℎ = 𝑃𝑎 + 𝜌𝑔ℎ
Where 𝑃ℎ – hydrostatic pressure
𝑃𝑎 – atmospheric pressure
𝜌 – density of fluid
g – gravity
𝜌 – height of fluid

This is also used in computing pressure variation with the depth in a


static fluid:

𝑃2 = 𝑃1 + 𝜌𝑔ℎ
Where 𝑃1 – pressure at point 1
𝑃2 – pressure at point 2

Note: The atmosphere is a deep ‘ocean’ of air which also exerts pressure
on us. At sea level, air pressure is about 1.0 x 105 N/m2 or 100 KPa

Sample Problems____________________________________________________

1. An observer watches a fish tank and notices that fish no.1 likes to feed at the
water’s surface while fish no.2 feeds at 0.459 m below the surface of the
water. If the average density of the water in the tank is 1000 kg/m3, what is
the pressure on fish no.2?

Given: P1 = 1.0 x 105 Pa


𝜌 = 1000 kg/m3
g = 9.8 m/s2
h = 0.459 m
Required: P2
Solution:
𝑃2 = 𝑃1 + 𝜌𝑔ℎ
kg m
𝑃2 = 1.0 x 105 Pa + (1000 m3 ) (9.8 s2 ) (0.459 m)
𝑷𝟐 = 𝟏. 𝟎𝟒 𝐱 𝟏𝟎𝟓 𝑷𝒂

2. The Mariana Trench, which is also known as the Philippine Deep measures
10, 863 m depth. What is the pressure at the bottom of the trench? Use
specific gravity of sea water = 1.03.

Given: h = 10,863 m
3 | Physics for Engineers 142

SG = 1.03
Required: 𝑃
Solution:
𝝆𝒍𝒊𝒒𝒖𝒊𝒅
𝑆𝐺𝑙𝑖𝑞𝑢𝑖𝑑 =
𝝆𝒘𝒂𝒕𝒆𝒓
𝜌𝑙𝑖𝑞𝑢𝑖𝑑
1.03 = 𝑘𝑔
1000
𝑚3
𝑘𝑔
𝜌𝑙𝑖𝑞𝑢𝑖𝑑 = 1030 𝑚3

Since we are asked for the pressure at the bottom of the water, we use the
formula for gauge pressure:

P = ρgh
= (1030 kg/m3)(9.8 m/s2)(10,863 m)
P = 1.0965 x 105 N/m2 or 1.0965 x 105 Pa

Pascal’s Principle: Transmitting Pressure


We may never be aware of it, but Pascal’s principle can be observed in our daily
activities. When you squeeze a tube of toothpaste, you demonstrate Pascal’s
principle. The pressure on you fingers exert at the bottom of the tube is
transmitted to the toothpaste, forcing the paste out of the opening. Pascal’s
principle applies to all fluids. A typical example of this is automobile lifts in
many cars services station.

Figure 10.1: Hydraulic Lift


Source: Hydraulic Lift (http://hydraulicliftdoketsue.blogspot.com/2017/03/pascal-law-
hydraulic-lift.html?m=1)

Consider Figure 10.1, in an automobile lift the force F1 exerted in point 1


provides pressure in the fluid and increasing the pressure inside resulting a
lifting force F2 in the throughput at point 2. The pressure applied to a confined
fluid increases the pressure throughput by the same amount. In this example,
the proportions are as follows:
3 | Physics for Engineers 143

𝑃1 = 𝑃2
𝐹1 𝐹2
=
𝐴1 𝐴2
𝐴2
𝐹2 = ( )𝐹
𝐴1 1
Sample Problem______________________________________________________

3. If a barber’s chair weighs 1800 N and rest on a large piston with a cross-
sectional area of 1500 cm2, what force must be applied to the smaller
piston with a cross-sectional area of 80 cm2 to lift the chair?

Given: F2 = 1800 N

A2 = 1500 cm2

A1 = 80 cm2

Required: F1
𝐹1 𝐹
Solution: = 𝐴2
𝐴1 2

𝐹2 𝐴1
𝐹1 = 𝐴2
(1800𝑁)(80 cm2)
𝐹1 = (1500 cm2)

𝑭𝟏 = 𝟗𝟔 𝑵
4. A 20 N force is exerted on the small piston of a hydraulic system shown
in the following diagram. The cross-section area of the small piston is 5 x
10-2 m2. What is the magnitude of the weight that can be lifted by the large
piston if it has a surface are of 1 x 10-1 m2?

Given: F2 = 20 N

A2 = 5 x 10-2 m2

A1 = 1 x 10-1 m2

Required: F1
𝐹1 𝐹
Solution: = 𝐴2
𝐴1 2

𝐹2 𝐴1
𝐹1 =
𝐴2
(20𝑁)(1 x 10−1 m2 )
𝐹1 =
(5 x 10−2 m2 )
𝑭𝟏 = 𝟒𝟎 𝑵
3 | Physics for Engineers 144

10.2.3. Buoyancy
There are many instances that occurs in our environment that sometimes
puzzles us like why would a coin sinks in water while a ferry boat float? Or
when a wood and a metal of the same shape and size are submerged in water,
the wood floats while the metal sinks.

Obviously, weight is not the reason. Object floats or sinks because of their
density not because of their weight. An object immersed in a fluid will sink if
the object is denser than the fluid. It will float if it is less dense than the fluid.

Now, let us focus on objects that somewhat floats. A term called Buoyancy
takes place when an object floats in a fluid. Buoyancy is the apparent loss of
weight when an object is immersed in a fluid because of the difference in
pressure that is exerted on all sides. It occurs because the pressure at the bottom
is larger than the pressure on the top surface. As a result, the fluid exerts a net
upward force called buoyant force.

Consider a buoyant object totally immersed in a fluid at rest and of constant


mass and density shown in Figure 10.2. The pressures (gauge) of the upper and
lower surfaces are as follows since it is in a fluid:
𝑃1(𝑡𝑜𝑝) = 𝜌𝑔ℎ1

𝑃2(𝑏𝑜𝑡𝑡𝑜𝑚) = 𝜌𝑔ℎ2

Figure 10.2: Calculating Buoyant Force Exerted on a Cube


Source: Padua, A.L. & Crisostomo, R. M. (2010). Science and Technology IV Practical and
Explorational Physics Modular Approach, 2nd Edition

The forces at the top and bottom are as follows:


𝐹𝑡𝑜𝑝 = 𝑃1(𝑡𝑜𝑝)𝐴 = 𝜌𝑔ℎ1 𝐴
𝐹𝑏𝑜𝑡𝑡𝑜𝑚 = 𝑃2(𝑏𝑜𝑡𝑡𝑜𝑚) 𝐴 = 𝜌𝑔ℎ2 𝐴

The net upward force is the buoyant force, 𝑭𝒃 ,


𝐹𝑛𝑒𝑡 = 𝐹𝑏
= 𝐹𝑏𝑜𝑡𝑡𝑜𝑚 − 𝐹𝑡𝑜𝑝
3 | Physics for Engineers 145

= 𝜌𝑔ℎ2 𝐴 − 𝜌𝑔ℎ1 𝐴
= 𝜌𝑔𝐴(ℎ2 − ℎ1 )
The volume of the solid is 𝑉𝑠𝑜𝑙𝑖𝑑 = 𝐴𝐿 and 𝐿 = ℎ2 − ℎ1 , therefore,

𝑉𝑜𝑏𝑗𝑒𝑐𝑡 = 𝐴(ℎ2 − ℎ1 )

Substituting the 𝑉𝑠𝑜𝑙𝑖𝑑 to the 𝐹𝑏 ,

𝐹𝑏 = 𝜌𝑔𝐴(ℎ2 − ℎ1 )
𝐹𝑏 = 𝜌𝑔𝑉𝑜𝑏𝑗𝑒𝑐𝑡

Since, by Archimedes’ Principle shown in Figure 10.3, also means that,


𝑉𝑑𝑖𝑠𝑝𝑙𝑎𝑐𝑒𝑑 𝑓𝑙𝑢𝑖𝑑 = 𝑉𝑠𝑢𝑏𝑚𝑒𝑟𝑔𝑒𝑑 𝑜𝑏𝑗𝑒𝑐𝑡

This also means that:

𝐹𝑏 = 𝜌𝑔𝑉𝑑𝑖𝑠𝑝𝑙𝑎𝑐𝑒𝑑 𝑓𝑙𝑢𝑖𝑑

Also, 𝜌𝑉𝑑𝑖𝑠𝑝𝑙𝑎𝑐𝑒𝑑 𝑓𝑙𝑢𝑖𝑑 = 𝑚𝑑𝑖𝑠𝑝𝑙𝑎𝑐𝑒𝑑 𝑓𝑙𝑢𝑖𝑑 ,

Therefore, we can express the buoyant force as:


𝑭𝒃 = 𝒎𝒅𝒊𝒔𝒑𝒍𝒂𝒄𝒆𝒅 𝒇𝒍𝒖𝒊𝒅(𝒈)

Where,

𝐹𝑏 = buoyant force

𝑔 = pull of gravity

𝑚𝑑𝑖𝑠𝑝𝑙𝑎𝑐𝑒𝑑 𝑓𝑙𝑢𝑖𝑑 = mass of the fluid displaced by the object

Figure 10.3: Archimedes’ Principle of Buoyancy


A 5-kg object immersed in water is shown being acted upon by a buoyant (upward) force of 2-kg, which
is equal to the weight of the water displaced by the immersed object. The buoyant force reduces the
objects’ apparent weight by 2-kg – that is from 5-kg to 3-kg.
Source: Encyclopedia Britannica, Inc.
3 | Physics for Engineers 146

▪ If the buoyant force is EQUAL to the weight of the object submerged in


the fluid, the object will REMAIN AT ANY LEVEL in the fluid.
▪ If the buoyant force is GREATER than the weight of the object submerged
in the fluid, the object will FLOAT.
▪ If the buoyant force is LESS than the weight of the object submerged in
the fluid, the object will SINK.

The buoyant force is greater in denser liquids than in less dense one. This is
also the reason why we float in seawater than in fresh water because seawater
(𝜌 = 1030 𝑘𝑔/𝑚3 ) is denser than freshwater (𝜌 = 1000 𝑘𝑔/𝑚3 ).

10.2.4. Surface Tension


Surface tension is the tendency of the surface of a liquid to contact in area and
behave like an elastic membrane. Common examples are dew drops, soap
bubbles, paper clip floating on water, and an insect walking on water surface.
This phenomenon happens because the outer layer of the liquid molecules is
spaced more widely than the inner layers.

Consider Figure 10.4 below, shows how a molecule behaves in a liquid surface.
The molecules on the surface do not experience any interaction from above.

As a result, the forces between them and the inner molecules tend to pull them
inwards and towards each other. This makes the surface appear like in a state
of tension. Surface tension from the word itself is basically caused by the
attractive forces between molecules at the surface.

Figure 10.4: A molecule at the surface is pulled only sideways and downward by neighboring
molecules and the molecule on the surface is pulled equally in all directions
Source: Padua, A.L. & Crisostomo, R. M. (2010). Science and Technology IV Practical and
Explorational Physics Modular Approach, 2nd Edition
3 | Physics for Engineers 147

Cohesion and Adhesion

Molecular attraction may be classified into two types - cohesion and adhesion.
Cohesion is the attraction of molecules of the same substance, whereas attraction
between different substances is called adhesion.

Adhesion and cohesion best observed in water and mercury placed inside a
separate glass tubes shown in Figure 10.5. The strong attraction between
molecules of water and the glass gives the water an upward curving surface, or
meniscus, where it meets the glass. On the other hand, the meniscus of mercury
curves downward where it meets the glass because the cohesion of the
molecules of mercury is stronger than their adhesion to glass. Surface tension is
an example of cohesion at the surface of the water.

Figure 10.5: A tube of water (dyed red) has concave meniscus because of adhesive forces
dominate (A).
A tube of mercury has a convex meniscus because of the strong cohesive forces between
molecules (B.)
Source: Padua, A.L. & Crisostomo, R. M. (2010). Science and Technology IV Practical and
Explorational Physics Modular Approach, 2nd Edition

Capillarity
Capillarity or Capillary Action is the elevation or depression of liquids in
small-diameter tubes. It is important in a lot of cases. It is the method by which
water rises in the stems of the plants to reach the other parts. It also assists the
pumping action of the heart to help keep blood moving in the blood vessels.
Capillary action is also the principle behind the rising of oil in the wick of an
oil lamp, absorption of water by paper towels and rising of paint through the
bristles of paintbrush.

Capillarity or capillary action depends on both adhesion and surface tension.


Adhesion causes water to go up the glass walls of the small-diameter tubes
and produce concave surface while surface tension tends to flatten this surface
3 | Physics for Engineers 148

by contraction. The combined action of these two forces raises the water above
the surrounding level. The water level rises until the upward force is counter-
balanced by the weight of the elevated liquid.

10.2.5. Fluid Dynamics and Bernoulli’s Principle


Fluids in motion often behave in complex and unpredictable ways. A basic
description of fluid flow is conveniently obtained by considering an ideal fluid.
An ideal fluid has zero viscosity (i.e., it offers no resistance to motion), is
incompressible (i.e., its density remains constant), and has a steady flow.
An ideal fluid undergoes two types of fluid flow – the streamline or laminar
flow and the turbulent flow. See Figure 10.6.

Fluid Flow Description


Streamline or The motion of the particle of fluid
Laminar flow passing a particular point is the
same as the motion of the particle
that preceded it. The path that the
particle takes is called streamline.
This occurs when water flows
smoothly in rivers through pipe,
and when the wind blows gently.
Turbulent flow It is marked by irregular motion of fluid
as it passes over an obstacle, setting up
whirlpools and eddies. The motion of a
particle after it passes a particular point
may be different from the motion of
particles that preceded it at that point.
This is exhibited by flowing water in a
river that hits with a great rock.
Turbulent flow is exhibited by any fluid
moving at high speeds.
3 | Physics for Engineers 149

Figure 10.6: Laminar and Turbulent Flow around an obstacle


Source: Padua, A.L. & Crisostomo, R. M. (2010). Science and Technology IV Practical and
Explorational Physics Modular Approach, 2nd Edition

When a fluid flows through a constriction, its velocity increases. You will
observe this if you stand on the bank of a river where the water current flows
with a higher velocity at the narrow portion of the river and slower at a wider
portion. And there is a sharp curve or narrow passage, the flow is turbulent.

The velocity of an incompressible fluid such as water can be measured by using


a Venturi tube. A Venturi tube consists of a plain tube with a smooth
constriction in its bore at the middle (Figure 10.7). It is used in carburetors, fluid
flow meters, and aircraft speed indicators. The levels in the vertical tubes
indicate the pressure of the fluid as it passes through the constricted tube. When
the velocity of the fluid increases, the pressure drops. The relationship between
fluid and pressure exerted by a moving fluid is described by Bernoulli’s
Principle, named after a Swiss mathematician, Daniel Bernoulli (1700-1782). It
states that, as the velocity of the fluid increases, its pressure decreases.

Figure 10.7: Laminar and Turbulent Flow around an obstacle


Source: Padua, A.L. & Crisostomo, R. M. (2010). Science and Technology IV Practical and
Explorational Physics Modular Approach, 2nd Edition
3 | Physics for Engineers 150

Figure 10.8: Demonstrating the rate of flow of a fluid


Source: Padua, A.L. & Crisostomo, R. M. (2010). Science and Technology IV Practical and
Explorational Physics Modular Approach, 2nd Edition

Figure 10.9: In a laminar flow, speed increases as the cross-sectional area of the pipe decreases
Source: Padua, A.L. & Crisostomo, R. M. (2010). Science and Technology IV Practical and
Explorational Physics Modular Approach, 2nd Edition

Applications of Bernoulli’s Principle

Ever wondered why a kite fly? A kite will only fly if there is a wind blowing
against it. Bernoulli’s principle accounts for the flight of the kite. Air meeting
the kite at launching position divides into two streams – upper and lower
streams. The upper stream passes above a flying kite and the lower stream
passes below the kite. The upper stream travels faster than the lower stream.
The difference in pressure due to the difference in speed of the air above and
below the kite keeps the kite afloat.

You can demonstrate Bernoulli’s principle by blowing across the surface of a


sheet of paper. The air pressure is higher beneath the paper, so the paper moves
upward, towards the area with lower air pressure.
3 | Physics for Engineers 151

Figure 10.10: The piece of paper experiences lifts as explain by Bernoulli’s Principle
Source: Padua, A.L. & Crisostomo, R. M. (2010). Science and Technology IV Practical and
Explorational Physics Modular Approach, 2nd Edition

The same principle is applied to make airplanes fly and to hold the rear wheels
of race cars on the road at high speeds. The top of an airplane wing is curved.
Air that moves over the top of the wing must travel farther than the air that
moves along the bottom of the wing. As a result, the air moving at the top
moves faster and exerts less pressure than the air at the bottom. The difference
is pressure creates an upward force on the wing, called the lift, which holds the
airplane upward.

On the other hand, race cars have spoilers on the back which is curved on the
lower side. The greater the pressure pushing downward on the spoiler gives
the car better traction from its rear wheels.

Figure 10.11: Airfoils, such as wings of airplanes, are designed to produce lift when moving
through a fluid (A). Spoilers act like inverted airfoils (B).
Source: Padua, A.L. & Crisostomo, R. M. (2010). Science and Technology IV Practical and
Explorational Physics Modular Approach, 2nd Edition
3 | Physics for Engineers 152

Assessment
Answer the two assessments below (Exercise and Quiz). Write/Encode the
assessments in a long bond paper.

You will be graded based on the following rubrics:


RUBRIC

A. Essay/Quiz

Criteria 5 4 3 2 1
The student was The student
able to elicit ideas The student is unable to
and correct was not only elicit ideas
The student
able to elicit and concepts
understanding was able to The student
ideas and from the
from the reading, elicit ideas and was able to
Understanding correct reading
shows evidence of correct elicit ideas
40% understanding indicating
internalizing these, understanding from the
but also shows that he/she
and contributed from the reading
evidence of has not read
additional thoughts reading.
internalizing the
to the core idea. these. prescribed
reading.
Shows correct Shows
Shows correct
content with Shows irrelevant and
content with
Content additional content Shows correct partial erroneous
evidence
40% resources not content. erroneous content. Did
sourced from
limited to the content. not follow
the module.
module. directions.
Submitted Submitted
Submitted
Submitted on or Submitted after after after
Timeliness after the
before the the deadline: deadline: deadline:
20% deadline:
deadline. 1 day – 3 days 9 days – 15 16 days - 30
4 days – 8 days
days days.
Note for timeliness: 0 points for late submission more than 30 days from the deadline.

Exercise No. 10
1. Which has more pressure, the bottom of the lake 7 meters deep or the bottom of the river 2
meters hallower than the lake but holds more volume of water?
2. We know that the atmosphere exerts pressure on all things. How come this atmospheric pressure
does not break glass windows?
3. How does water pressure 1 meter below the surface of a small pond compare with the water
pressure at the same distance below the surface of the lake?
4. Will the plastic box that is filled with tennis ball sink deeper into the water than an empty plastic
box of the same size and shape? Explain.
5. Why do water and alcohol cling to the surface of a glass rod whereas mercury does not?
6. What causes the roofs if houses lifted and blown away by strong winds?
3 | Physics for Engineers 153

Quiz No. 10

1. Ricky is raising a 1200-kg car on his hydraulic lift so that he can work underneath. If the area of
the input piston is 12 cm2 while the output piston is 700 cm2, what force must be exerted on the
input piston to lift the car?
2. A container is filled with water to a depth of 20 cm. On top of the water floats a 30-cm thick layer
of oil with a density of 0.70x103 kg/m3. What is the pressure at the surface of the water? What is
the absolute pressure at the bottom of the container?
3. A cube of steel 1.0 dm3 in volume is immersed in water. What is the magnitude of the buoyant
force acting on it? Density of steel is 9.0 x 103 kg/m3
4. With the same problem in Item No.3, what will be the net weight of the cube?
5. Dentist’s chairs are examples of hydraulic lifts. If the chair weighs 16 x 102 N and rests on a
piston with cross-sectional area of 14.4 x 102 cm2, what force must be applied to the small piston
with a cross-sectional area of 72 cm2 to lift the chair?
6. A dam springs a leak through a hole of 1 cm3 at a depth of 2 cm below the water surface. With
what force would a boy have to push on the hole with his thumb to stop the leak? Could he stop
the leak?

10.3. References
Padua, A.L. & Crisostomo, R. M. (2010). Science and Technology IV Practical and
Explorational Physics Modular Approach, 2nd Edition, Philippines: Vibal
Publishing House Inc.

Young, H.D. (1992). University Physics, 8th Edition, Addison-Wesley Pub. Co.
Review of Simple Harmonic Motion. Flipping Physics. March 23, 2020.
https://www.flippingphysics.com/ap1-shm-review.html.
Collection: OpenStax University Physics - All Volumes Edited by: Cengage
WebAssign URL: https://legacy.cnx.org/content/col12105/1.1/ Copyright:
Cengage WebAssign License: http://creativecommons.org/licenses/by/4.0/
Based on: University Physics <http://legacy.cnx.org/content/col11994/1.1>
arranged by OpenStax University Physics.

Quiseo, J.M. (2012). General Engineering & Applied Sciences Source Book 2nd
Edition. Philippines: GioBooks

https: byjus.com/physics/types-of-waves

Del Rosario, A.C.M. (2004). College Physics.Philippines

Encyclopedia Britannica, Inc.

Hydraulic Lift (http://hydraulicliftdoketsue.blogspot.com/2017/03/pascal-


law-hydraulic-lift.html?m=1)
3 | Physics for Engineers 154

10.4. Acknowledgement

The images, tables, figures, and information contained in this module were
taken from the references cited above.
3 | Physics for Engineers 155

UNIT 11: HEAT TRANSFER

11.0. Intended Learning Outcomes


By the end of this unit, you should be able to:
a. Describe three methods of heat transfer and solve basic problems in
each method.
b. Identify the three forms of heat transfer.
c. Explain some phenomena that involve conductive, convective, and
radiative heat transfer.
d. Solve problems on the relationships between heat transfer, time, and
rate of heat transfer.
e. Solve problems using the formulas for conduction and radiation.

11.1. Introduction
A The study of heat flow within an object and from one object to another due
to differences in temperature is known as heat transfer. Whenever there is a
temperature difference, heat transfer occurs. It may occur rapidly, as through a
cooking pan, or slowly, as through the walls of a picnic ice chest. So many
processes involve heat transfer that it is hard to imagine a situation where no
heat transfer occurs. Yet every heat transfer takes place by only three methods:

1. Conduction is heat transfer through stationary matter by direct physical


contact. (The matter is stationary on a macroscopic scale—we know that
thermal motion of the atoms and molecules occurs at any temperature
above absolute zero.) Heat transferred from the burner of a stove through
the bottom of a pan to food in the pan is transferred by conduction.
2. Convection is the heat transfer by the macroscopic movement of a fluid.
This type of transfer takes place in a forced-air furnace and in weather
systems, for example.
3. Heat transfer by radiation occurs when microwaves, infrared radiation,
visible light, or another form of electromagnetic radiation is emitted or
absorbed. An obvious example is the warming of Earth by the Sun. A less
obvious example is thermal radiation from the human body.

In this unit, we will examine the three methods better in more detail. Each
method has unique and interesting characteristics, but all three have two things
in common:

1. They transfer heat solely because of a temperature difference,


2. and the greater the temperature difference, the faster the heat transfer.
3 | Physics for Engineers 156

Figure 11.1: Conduction, Convection & Radiation


Source: Moses, Ntam, Calculus Based General Physics 2 (PHYS 311). OpenStax CNX. Dec
20, 2019. http://cnx.org/contents/9c45112c-cc2e-4e89-8f91-5e3c25cd65f2@1.140.

Consider Figure 11.1, in a fireplace, heat transfer occurs by all three


methods: conduction, convection, and radiation. Radiation is responsible for
most of the heat transferred into the room. Heat transfer also occurs through
conduction into the room, but much slower. Heat transfer by convection also
occurs through cold air entering the room around windows and hot air leaving
the room by rising the chimney.

11.2. Discussions and Assessments


11.2.1. Calorimetry: Calculations of Heat
Calorimetry is the science of measuring heat in a chemical or physical process.
This study observes the temperature change of a substance when it absorbs or
release heat.

Some substance requires a lot of heat energy to raise their temperature by


1℃ 𝑜𝑟 1𝐾. These substances are suitable as coolants like water H2O. Metals like
copper, iron, and zinc on the other hand, are considered as substances that
doesn’t require much of heat energy. These types of substances are quite easy
to raise their temperature by 1℃ 𝑜𝑟 1𝐾.

Now let us consider three concepts useful in understanding temperature


change and heat flow:

▪ Heat Capacity (symbol: C) - is the quantity of heat (q) that is absorbed


or released when a substance experiences a temperature change (ΔT) of
1℃ 𝑜𝑟 1𝐾.
3 | Physics for Engineers 157

𝑞 𝑞
𝐶= =
∆𝑇 (𝑇𝑓𝑖𝑛𝑎𝑙 − 𝑇𝑖𝑛𝑖𝑡𝑖𝑎𝑙 )

𝑞 = 𝐶∆𝑇 = 𝐶(𝑇𝑓𝑖𝑛𝑎𝑙 − 𝑇𝑖𝑛𝑖𝑡𝑖𝑎𝑙 )

Where,
C = Heat Capacity (SI unit: J/K, English unit: BTU/R)
q = Heat absorbed or released (SI unit: J, English unit: BTU)
∆𝑇 = Change in Temperature (SI unit: K, English unit: R)
*note that K is kelvin and R is Rankine, both are absolute temperatures.

The higher the heat capacity, the smaller the change in temperature for a
given amount of heat that is absorbed. Water has high heat capacity
than metals. This is the reason that water is a better coolant than metals.

▪ Specific Heat Capacity (symbol: c) – commonly known as specific heat


is the quantity of heat (q) required to raise 1g of a substance by 1℃ 𝑜𝑟 1𝐾.

𝑞 𝑞
𝑐= =
𝑚∆𝑇 𝑚(𝑇𝑓𝑖𝑛𝑎𝑙 − 𝑇𝑖𝑛𝑖𝑡𝑖𝑎𝑙 )

𝑞 = 𝑚𝑐∆𝑇 = 𝑚𝑐(𝑇𝑓𝑖𝑛𝑎𝑙 − 𝑇𝑖𝑛𝑖𝑡𝑖𝑎𝑙 )

It can be modified if the given is in moles, you may use the molar heat
capacity:
𝑞 = 𝑛𝑐∆𝑇 = 𝑛𝑐(𝑇𝑓𝑖𝑛𝑎𝑙 − 𝑇𝑖𝑛𝑖𝑡𝑖𝑎𝑙 )

Where,
c = Specific Heat Capacity (SI unit: J/gK, English unit: BTU/lbR)
m = mass of the substance (SI unit: kg or g, English unit: lb)
n = number of moles of the substance (SI unit: mol)
q = Heat absorbed or released (SI unit: J, English unit: BTU)
∆𝑇 = Change in Temperature (SI unit: K, English unit: R)
*note that K is kelvin and R is Rankine, both are absolute temperatures

Remember that heat capacity (C) does not mention the amount of substance
that is being considered while specific heat capacity (c) does.

Values of heat capacity and specific heats of common substances are present in
Appendix A (attached at the end of this unit). Specific heat capacity can also
be expressed in moles instead of grams, as presented in Appendix A, this comes
in handy when the given substance is expressed in moles.
3 | Physics for Engineers 158

▪ Thermal Equilibrium

When two objects with different temperatures are placed in contact,


heat flows from the one with higher temperature to the one with cooler
temperature (hot → cold) until both objects reached an equal
temperature. When this happens the two objects is said to reached
thermal equilibrium.

contact
Hot object (1) Cool object (2)
T (1) initial T (2) initial

Heat loss
Thermal Heat gain
Equilibrium
T final

©nojuska123/favpng.com
𝑇 = 70℃

𝑇 = −10℃

𝑇𝑒𝑞𝑢𝑖𝑙𝑖𝑏𝑟𝑖𝑢𝑚 = 𝑇𝑓𝑖𝑛𝑎𝑙 = 40℃

As stated in the law of conservation of energy, the total energy is


constant. Since we are talking about the change in total heat energy, the
equation will be:
∆𝑞 = 0
𝑞𝑙𝑜𝑠𝑡 + 𝑞𝑔𝑎𝑖𝑛 = 0
𝑞𝑙𝑜𝑠𝑡 = −𝑞𝑔𝑎𝑖𝑛
(𝑚𝑐∆𝑇)ℎ𝑜𝑡 = −(𝑚𝑐∆𝑇)𝑐𝑜𝑙𝑑

The amount of energy lost by the hotter object is the amount of energy
gained by the cooler one. Thus, the total energy is still the same
consistent with the law of conservation of energy.
3 | Physics for Engineers 159

Example: Measuring Heat____________________________________________


1. Heating a 33.0-g aluminum can raises its temperature by 12.0°C. Find
the value of q for the can.

Given: m = 33 g
∆𝑇 = 12.0°C = 12.0 K
c = 0.9 J/g K (Appendix A)
Required: q
Solution:
𝑞 = 𝑚𝑐∆𝑇

Note that ∆𝑇 𝑖𝑛 𝑐𝑒𝑙𝑐𝑖𝑢𝑠 (℃) = ∆𝑇 𝑖𝑛 𝑘𝑒𝑙𝑣𝑖𝑛 (𝐾),


however, 𝑇 𝑖𝑛 ℃ ≠ 𝑇 𝑖𝑛 𝐾. Only the difference/change temperature is equal.

𝐽
𝑞 = (33 𝑔) (0.9 ) (12 𝐾)
𝑔∙𝐾
𝒒 = 𝟑𝟓𝟔. 𝟒 𝑱

2. The molar heat capacity of liquid water is 75.3 J/mol K. If 28.5 g of water
is cooled from 32.0 to 5.0°C, what is q for the water?

Given: m = 28.5 g
𝑇𝑖𝑛𝑖𝑡𝑖𝑎𝑙 = 32.0°C
𝑇𝑓𝑖𝑛𝑎𝑙 = 5.0°C
c = 75.3 J/mol K
Required: q
Solution:

Notice that the given specific heat is in mol and the water is in grams. So, we
need the molar mass of water to convert the given mass into moles:

For H2O: 1(atomic mass of H) + 2(atomic mass O)


2(1.008 g/mol) + 1(16.0 g/mol)
= 18.016 g/mol
Now let us convert the mass into moles,
1 𝑚𝑜𝑙
28.5 𝑔 ( ) = 1.583 𝑚𝑜𝑙
18.016 𝑔
Let us convert the temperatures into kelvin,
T in K = [(T in °C) + 273.15] K
𝑇𝑖𝑛𝑖𝑡𝑖𝑎𝑙 = [32.0 + 273.15]K = 305.15 K
𝑇𝑓𝑖𝑛𝑎𝑙 = [5.0 + 273.15]K = 278.15 K
3 | Physics for Engineers 160

∆𝑇 = 𝑇𝑖𝑛𝑖𝑡𝑖𝑎𝑙 − 𝑇𝑓𝑖𝑛𝑎𝑙 = 305.15 𝐾 − 278.15 𝐾 = 27𝐾


Let us prove if ∆𝑇 𝑖𝑛 𝐾 𝑖𝑠 𝑟𝑒𝑎𝑙𝑙𝑦 𝑒𝑞𝑢𝑎𝑙 𝑡𝑜 ∆𝑇 𝑖𝑛 ℃,
∆𝑇 = 𝑇𝑖𝑛𝑖𝑡𝑖𝑎𝑙 − 𝑇𝑓𝑖𝑛𝑎𝑙 = 32.0℃ − 5℃ = 27℃
Notice that both has value of 27, therefore, ∆𝑇 𝐾 = ∆𝑇℃

Now let us compute for the heat (q):


𝑞 = 𝑛𝑐∆𝑇
𝐽
𝑞 = (1.538 𝑚𝑜𝑙 ) (75.3 ) (27𝐾)
𝑚𝑜𝑙 ∙ 𝐾
𝒒 = 𝟑, 𝟏𝟐𝟔. 𝟗 𝑱

3. A glass contains 350.0 g of warm water at 75.0°C. A piece of gold at


3.80°C is placed in the water. The final temperature reached by this
system is 66.2°C. What was the mass of gold? The specific heat of water
is 4.184 J/g °C, and that of gold is 0.129 J/g °C.

Given:

𝑇𝐴𝑢 = 3.80℃ = 276.95𝐾


𝑚𝐴𝑢 = ? 𝑔
𝐽
𝑇𝐻2𝑂 = 75℃ = 348.15𝐾 𝑐𝐴𝑢 = 0.129
𝑔∙𝐾
𝑚𝐻20 = 350 𝑔
𝐽
𝑐𝐻20 = 4.184
𝑔∙𝐾

𝑇𝑒𝑞𝑢𝑖𝑙𝑖𝑏𝑟𝑖𝑢𝑚 = 𝑇𝑓𝑖𝑛𝑎𝑙 = 66.2℃ = 339.35𝐾

Required: m Au
Solution:
∆𝑞 = 0
𝑞𝑙𝑜𝑠𝑡 + 𝑞𝑔𝑎𝑖𝑛 = 0
𝑞𝑙𝑜𝑠𝑡 = −𝑞𝑔𝑎𝑖𝑛
(𝑚𝑐∆𝑇)ℎ𝑜𝑡 = −(𝑚𝑐∆𝑇)𝑐𝑜𝑙𝑑
(𝑚𝑐∆𝑇)𝐻20 = −(𝑚𝑐∆𝑇)𝐴𝑢
𝐽 𝐽
(350 𝑔) (4.184 ) (339.35 𝐾 − 348.15 𝐾) = −(𝑚)𝐴𝑢 (0.129 ) (339.35𝐾 − 276.95𝐾)
𝑔∙𝐾 𝑔∙𝐾
𝐽
−12,886.72 𝐽 −𝑚𝐴𝑢 (8.0496 𝑔 )
=
𝐽 𝐽
−8.0496 −8.0496
𝑔 𝑔
1,600.92 𝑔 = 𝑚𝐴𝑢
𝒎𝑨𝒖 = 𝟏, 𝟔𝟎𝟎. 𝟗𝟐 𝒈
3 | Physics for Engineers 161

11.2.2. Conduction
As you walk barefoot across the living room carpet in a cold house and then
step onto the kitchen tile floor, your feet feel colder on the tile. This result is
intriguing, since the carpet and tile floor are both at the same temperature. The
different sensation is explained by the different rates of heat transfer: The heat
loss is faster for skin in contact with the tiles than with the carpet, so the sensation of
cold is more intense.

Some materials conduct thermal energy faster than others. Fiberglass batt used
in houses is a material that conducts heat slowly—it is a good thermal insulator,
or poor heat conductor—used to reduce heat flow into and out of a house.

There are four factors that affect conduction:


1. Temperature Difference
2. Cross-sectional Area
3. Thickness
4. Thermal Conductivity

These factors are used as a basis in determining the rate of heat transfer by
conduction. A molecular picture of heat conduction will help justify the
equation that describes it. Figure 11.2 shows molecules in two bodies at
different temperatures, Th and Tc, for “hot” and “cold.” The average kinetic
energy of a molecule in the hot body is higher than in the colder body. If two
molecules collide, energy transfers from the high-energy to the low-energy
molecule.

Figure 11.2: Molecules in two bodies at different temperatures have different average kinetic energies.
Collisions occurring at the contact surface tend to transfer energy from high temperature regions to low-
temperature regions. In this illustration, a molecule in the lower-temperature region (right side) has low energy
before collision, but its energy increases after colliding with a high-energy molecule at the contact surface. In
contrast, a molecule in the higher-temperature region (left side) has high energy before collision, but its energy
decreases after colliding with a low-energy molecule at the contact surface.
Source: Moses, Ntam, Calculus Based General Physics 2 (PHYS 311). OpenStax CNX. Dec
20, 2019. http://cnx.org/contents/9c45112c-cc2e-4e89-8f91-5e3c25cd65f2@1.140.
3 | Physics for Engineers 162

In a metal, the picture would also include free valence electrons colliding with
each other and with atoms, likewise transferring energy. The cumulative effect
of all collisions is a net flux of heat from the hotter body to the colder body.
Thus, the rate of heat transfer increases with increasing temperature difference
𝜟𝑻 = 𝑻𝒉 − 𝑻𝒄 . If the temperatures are the same, the net heat transfer rate is zero.
Because the number of collisions increases with increasing area, heat
conduction is proportional to the cross-sectional area—a second factor in the
equation.

A third quantity that affects the conduction rate is the thickness of the material
through which heat transfers. Figure 11.3 shows a slab of material with a higher
temperature on the left than on the right. Heat transfers from the left to the right
by a series of molecular collisions. The greater the distance between hot and
cold, the more time the material takes to transfer the same amount of heat.

The fourth quantity that affects conduction rate is the thermal conductivity.
Thermal conductivities (k) are dependent on the type of material. It is a constant
that is determined through experimentation. Table 11.1 shows thermal
conductivities of common substances.

Figure 11.3: Heat conduction occurs through any material, represented here by a rectangular bar, whether
window glass or walrus blubber.
Source: Moses, Ntam, Calculus Based General Physics 2 (PHYS 311). OpenStax CNX. Dec
20, 2019. http://cnx.org/contents/9c45112c-cc2e-4e89-8f91-5e3c25cd65f2@1.140.

All four of these quantities appear in a simple equation deduced from and
confirmed by experiments. The rate of conductive heat transfer through a slab
of material, such as the one in Figure 11.3, is given by

𝒅𝑸 𝒌𝑨(𝑻𝒉 − 𝑻𝒄 )
𝑷= = 𝑬𝒒. 𝟏𝟏. 𝟏
𝒅𝒕 𝒙

Where,
P → the power or rate of heat transfer in watts or in kilocalories per second
A → cross-sectional area
x → thickness, (it is “d” as shown in Figure 11.3)
3 | Physics for Engineers 163

Th−Tc → is the temperature difference across the slab


k → thermal conductivity of the material

More generally, we can write Eq.11.1 as:

𝒅𝑻
𝑷 = −𝒌𝑨 𝑬𝒒. 𝟏𝟏. 𝟐
𝒅𝒙

where x is the coordinate in the direction of heat flow. Since in Figure 11.3, the
power and area are constant, dT/dx is constant, and the temperature decreases
linearly from Th to Tc.

Table 11.1: Thermal Conductivities of Common Substance

*Values are given for temperatures near 0°C.


Source: Moses, Ntam, Calculus Based General Physics 2 (PHYS 311). OpenStax CNX. Dec
20, 2019. http://cnx.org/contents/9c45112c-cc2e-4e89-8f91-5e3c25cd65f2@1.140.

Example: Conduction Calculation______________________________________


4. A polystyrene foam icebox has a total area of 0.950 m2 and walls with an
average thickness of 2.50 cm. The box contains ice, water, and canned
beverages at 0°C. The inside of the box is kept cold by melting ice. How
much ice melts in one day if the icebox is kept in the trunk of a car at
35.0ºC? 𝐻𝑓 = 334𝑥103 𝐽/𝑘𝑔.
3 | Physics for Engineers 164

Given: k = 0.010W/m·°C for polystyrene foam (Table 1)


A = 0.950m2
d = 2.50cm = 0.0250m
Tc = 0°C
Th = 35.0°C
t = 1day = 24hours = 86,400s.
Required: mass of ice, m ice
Solution:
This question involves both heat for a phase change (melting of ice) and the
transfer of heat by conduction. To find the amount of ice melted, we must find the
net heat transferred. This value can be obtained by calculating the rate of heat
transfer by conduction and multiplying by time.

The rate of heat transfer by conduction is given by:


𝒅𝑸 𝒌𝑨(𝑻𝒉 − 𝑻𝒄 )
𝑷= =
𝒅𝒕 𝒙
(0.010W/m · °C)(0.950m2 )(35.0°C − 0°C)
𝑃=
0.0250m
𝑃 = 13.3 𝑊
Since,
𝑑𝑄
𝑃=
𝑑𝑡
𝑄 = 𝑃𝑡 = (13.3𝑊 )(86,400s) = 1.15 × 106 J
The heat used to melt the is 𝑸 = 𝒎𝑯𝒇, where 𝐻𝑓 = 334𝑥103 𝐽/𝑘𝑔 for water

𝑸 = 𝒎𝑯𝒇

𝑄 1.15 × 106 J
𝑚= = = 𝟑. 𝟒𝟒 𝒌𝒈
𝐻𝑓 334𝑥103 𝐽/𝑘𝑔

The result of 3.44 kg, or about 7.6 lb, seems about right, based on experience. You
might expect to use about a 4 kg (7–10 lb) bag of ice per day. A little extra ice is
required if you add any warm food or beverages.

In developing insulation, the smaller the conductivity k and the larger the
thickness d, the better. Thus, the ratio d/k, called the R factor, is large for a
good insulator. The rate of conductive heat transfer is inversely
proportional to R. R factors are most commonly quoted for household
insulation, refrigerators, and the like.
3 | Physics for Engineers 165

Unfortunately, in the United States, R is still in non-metric units of


ft2·°F·h/Btu, although the unit usually goes unstated [1 British thermal
unit (Btu) is the amount of energy needed to change the temperature of 1.0 lb of
water by 1.0°F, which is 1055.1 J].

Note that in Table 11.1, most of the best thermal conductors—


silver, copper, gold, and aluminum—are also the best electrical
conductors, because they contain many free electrons that can transport
thermal energy. (Diamond, an electrical insulator, conducts heat by
atomic vibrations.) Cooking utensils are typically made from good
conductors, but the handles of those used on the stove are made from
good insulators (bad conductors).

5. A steel rod and an aluminum rod, each of diameter 1.00 cm and length
25.0 cm, are welded end to end. One end of the steel rod is placed in a
large tank of boiling water at 100°C, while the far end of the aluminum
rod is placed in a large tank of water at 20°C. The rods are insulated so
that no heat escapes from their surfaces. What is the temperature at the
joint, and what is the rate of heat conduction through this composite
rod?

Given: LAl = Lsteel = 0.25m


AAl = Asteel = 7.85×10−5 m2
kAl = 220 W/m·°C
ksteel = 80W/m·°C
Th = 100°C
Tc = 20°C.
Required: Temperature at the joint, Tjoint
Rate of heat conduction, P
The heat that enters the steel rod from the boiling water has no place to go but
through the steel rod, then through the aluminum rod, to the cold water.
Therefore, we can equate the rate of conduction through the steel to the rate of
conduction through the aluminum. We repeat the calculation with a second
method, in which we use the thermal resistance R of the rod, since it simply adds
when two rods are joined end to end.
Solution 1:

Calculate the heat-conduction rate through the steel rod and the heat-
conduction rate through the aluminum rod in terms of the unknown
temperature Tat the joint:
𝑑𝑄 𝑘𝐴(𝑇ℎ −𝑇𝑐 )
`𝑃= =
𝑑𝑡 𝑥
3 | Physics for Engineers 166

𝑘𝑠𝑡𝑒𝑒𝑙 𝐴𝑠𝑡𝑒𝑒𝑙 ∆𝑇𝑠𝑡𝑒𝑒𝑙


𝑃𝑠𝑡𝑒𝑒𝑙 =
𝐿𝑠𝑡𝑒𝑒𝑙
W
(80 m · °C) (7.85 × 10−5 m2 )(100°C − T𝑗𝑜𝑖𝑛𝑡 )
=
0.25 m
= (0.0251𝑊/°𝐶)(100°𝐶 − 𝑇𝑗𝑜𝑖𝑛𝑡 )

𝑘𝐴𝑙 𝐴𝐴𝑙 ∆𝑇𝐴𝑙


𝑃𝐴𝑙 =
𝐿𝐴𝑙
W
(220 m · °C) (7.85 × 10−5 m2 )(T𝑗𝑜𝑖𝑛𝑡 − 20°C)
=
0.25 m
= (0.0691𝑊/°𝐶)(𝑇𝑗𝑜𝑖𝑛𝑡 − 20°𝐶)

Set the two rates equal and solve for the unknown temperature:
𝑊 𝑊
(0.0691 ) (𝑇𝑗𝑜𝑖𝑛𝑡 − 20°𝐶) = (0.0251 ) (100°𝐶 − 𝑇𝑗𝑜𝑖𝑛𝑡 )
°𝐶 °𝐶
𝑻𝒋𝒐𝒊𝒏𝒕 = 𝟒𝟏. 𝟑 °𝑪

Calculate either rate:


𝑃𝑠𝑡𝑒𝑒𝑙 = (0.0251𝑊/°𝐶)(100°𝐶 − 41.3°𝐶) = 𝟏. 𝟒𝟕𝑾
𝑃𝐴1 = (0.0691𝑊/°𝐶)(41.3°𝐶 − 20°𝐶) = 𝟏. 𝟒𝟕𝑾

Solution 2:
𝐿 𝑘𝐴𝛥𝑇 𝐴𝛥𝑇 𝑃𝑅
Recall that 𝑅 = 𝑘. Now, 𝑃 = = , or 𝛥𝑇 =
𝐿 𝑅 𝐴

We know that 𝛥𝑇𝑠𝑡𝑒𝑒𝑙 + 𝛥𝑇𝐴𝑙 = 100°𝐶 − 20°𝐶 = 80°𝐶. We also know that
𝑃𝑠𝑡𝑒𝑒𝑙 = 𝑃𝐴𝑙 , and we denote that rate of heat flow by P. Combine the
equations:
𝑃𝑅𝑠𝑡𝑒𝑒𝑙 𝑃𝑅𝐴𝑙
+ = 80°𝐶
𝐴 𝐴
80°𝐶
𝑃=
𝐴(𝑅𝑠𝑡𝑒𝑒𝑙 + 𝑅𝐴𝑙 )
Find the 𝑅 from the known quantities:

𝑅𝑠𝑡𝑒𝑒𝑙 = 3.13 × 10−3 𝑚2 · °𝐶/𝑊


𝑅𝐴𝑙 = 1.14 × 10−3 𝑚2 · °𝐶/𝑊
3 | Physics for Engineers 167

Substitute the values to find P:


80°𝐶
𝑃=
𝐴(𝑅𝑠𝑡𝑒𝑒𝑙 + 𝑅𝐴𝑙 )
80°𝐶
=
(7.85 × 10−5 m2 )(3.13 × 10−3 𝑚2· °𝐶/𝑊 + 1.14 × 10−3 𝑚2 · °𝐶/𝑊)
= 𝟏. 𝟒𝟕𝑾
Determine 𝛥𝑇 for the aluminum rod (or for the steel rod) and use it to
find 𝑇 at the joint.

−3 2 °𝐶
𝑃𝑅𝐴𝑙 (1.47𝑊 ) (1.14 × 10 𝑚 · 𝑊 )
∆𝑇𝐴𝑙 = = = 21.3℃
𝐴 7.85 × 10−5 m2
So, 𝑇 𝑗𝑜𝑖𝑛𝑡 = 20°𝐶 + 21.3°𝐶 = 𝟒𝟏. 𝟑°𝑪, as in Solution 1.

In practice, adding R values is common, as in calculating the R value of


an insulated wall. In the analogous situation in electronics, the resistance
corresponds to AR in this problem and is additive even when the areas are
unequal, as is common in electronics. Our equation for heat conduction can be
used only when the areas are equal; otherwise, we would have a problem in three-
dimensional heat flow, which is beyond our scope.

Conduction is caused by the random motion of atoms and molecules. As such,


it is an ineffective mechanism for heat transport over macroscopic distances
and short times. For example, the temperature on Earth would be unbearably
cold during the night and extremely hot during the day if heat transport in the
atmosphere were only through conduction. Also, car engines would overheat
unless there was a more efficient way to remove excess heat from the pistons.

11.2.3. Convection
In convection, thermal energy is carried by the large-scale flow of matter. It can
be divided into two types:

1. In forced convection, the flow is driven by fans, pumps, and the like. A
simple example is a fan that blows air past you in hot surroundings and
cools you by replacing the air heated by your body with cooler air. A more
complicated example is the cooling system of a typical car, in which a
pump moves coolant through the radiator and engine to cool the engine
and a fan blows air to cool the radiator.
2. In free or natural convection, the flow is driven by buoyant forces: hot
fluid rises and cold fluid sinks because density decreases as temperature
3 | Physics for Engineers 168

increases. The house in Figure 11.4 is kept warm by natural convection, as


is the pot of water on the stove in Figure 11.5. Ocean currents and large-
scale atmospheric circulation, which result from the buoyancy of warm
air and water, transfer hot air from the tropics toward the poles and cold
air from the poles toward the tropics. (Earth’s rotation interacts with those
flows, causing the observed eastward flow of air in the temperate zones.)

Figure 11.4: Air heated by a so-called gravity furnace expands and rises, forming a convective loop that
transfers energy to other parts of the room. As the air is cooled at the ceiling and outside walls, it contracts,
eventually becoming denser than room air and sinking to the floor. An intelligently designed heating system using
natural convection, like this one, can heat a home quite efficiently.
Source: Moses, Ntam, Calculus Based General Physics 2 (PHYS 311). OpenStax CNX. Dec
20, 2019. http://cnx.org/contents/9c45112c-cc2e-4e89-8f91-5e3c25cd65f2@1.140.

Figure 11.5: Natural convection plays an important role in heat transfer inside this pot of water. Once conducted
to the inside, heat transfer to other parts of the pot is mostly by convection. The hotter water expands, decreases
in density, and rises to transfer heat to other regions of the water, while colder water sinks to the bottom. This
process keeps repeating.
Source: Moses, Ntam, Calculus Based General Physics 2 (PHYS 311). OpenStax CNX. Dec
20, 2019. http://cnx.org/contents/9c45112c-cc2e-4e89-8f91-5e3c25cd65f2@1.140.
3 | Physics for Engineers 169

Some interesting phenomena happen when convection is accompanied by a


phase change. The combination allows us to cool off by sweating even if the
temperature of the surrounding air exceeds body temperature. Heat from the
skin is required for sweat to evaporate from the skin, but without airflow, the
air becomes saturated, and evaporation stops. Airflow caused by convection
replaces the saturated air by dry air and evaporation continues.

Example: Convection Calculation______________________________________


6. The average person produces heat at the rate of about 120 W when at rest.
At what rate must water evaporate from the body to get rid of all this
energy? (For simplicity, we assume this evaporation occurs when a person
is sitting in the shade and surrounding temperatures are the same as skin
temperature, eliminating heat transfer by other methods.) H v = 2430kJ/kg
= 2430J/g.

Given: P = 120 W
Hv = 2430kJ/kg = 2430J/g.
Required: rate of water, m/t
Solution:

Energy is needed for this phase change (𝑸 = 𝒎𝑯𝑽 ). Thus, the energy loss
per unit time is,
𝑑𝑄
𝑃=
𝑑𝑡
𝑚𝐻𝑉
=
𝑡
= 120𝑊
= 120 𝐽/𝑠

We divide both sides of the equation by 𝐻𝑉 to find that the mass evaporated
per unit time is:

𝑚𝐻𝑉
= 120 𝐽/𝑠
𝑡
𝑚 120 𝐽/𝑠
=
𝑡 𝐻𝑉
𝑚 120 𝐽/𝑠
=
𝑡 2430J/g
𝑚
= 0.0494 𝑔/𝑠
𝑡
𝑚
= 𝟐. 𝟗𝟔 𝒈/𝒎𝒊𝒏
𝑡
3 | Physics for Engineers 170

Evaporating about 3 g/min seems reasonable. This would be about 180 g (about 7
oz.) per hour. If the air is very dry, the sweat may evaporate without even being
noticed. A significant amount of evaporation also takes place in the lungs and
breathing passages.

Another important example of the combination of phase change and


convection occurs when water evaporates from the oceans. Heat is removed
from the ocean when water evaporates. If the water vapor condenses in liquid
droplets as clouds form, possibly far from the ocean, heat is released in the
atmosphere. Thus, there is an overall transfer of heat from the ocean to the
atmosphere.

Figure 11.6: Cumulus clouds are caused by water vapor that rises because of convection. The rise of clouds is
driven by a positive feedback mechanism. (credit: “Amada44”/Wikimedia Commons)
Source: Moses, Ntam, Calculus Based General Physics 2 (PHYS 311). OpenStax CNX. Dec
20, 2019. http://cnx.org/contents/9c45112c-cc2e-4e89-8f91-5e3c25cd65f2@1.140.

This process is the driving power behind thunderheads, those great cumulus
clouds that rise as much as 20.0 km into the stratosphere (Figure 11.6). Water
vapor carried in by convection condenses, releasing tremendous amounts of
energy. This energy causes the air to expand and rise to colder altitudes. More
condensation occurs in these regions, which in turn drives the cloud even
higher. This mechanism is an example of positive feedback since the process
reinforces and accelerates itself. It sometimes produces violent storms, with
lightning and hail. The same mechanism drives hurricanes.
3 | Physics for Engineers 171

11.2.4. Radiation
You can feel the heat transfer from the Sun. The space between Earth and the
Sun is largely empty, so the Sun warms us without any possibility of heat
transfer by convection or conduction. Similarly, you can sometimes tell that the
oven is hot without touching its door or looking inside—it may just warm you
as you walk by.

In these examples, heat is transferred by radiation (Figure 11.7). That is, the hot
body emits electromagnetic waves that are absorbed by the skin. No medium
is required for electromagnetic waves to propagate. Different names are used
for electromagnetic waves of different wavelengths: radio waves, microwaves,
infrared radiation, visible light, ultraviolet radiation, X-rays, and gamma rays.

Figure 11.7: Most of the heat transfer from this fire to the observers occurs through infrared radiation. The
visible light, although dramatic, transfers relatively little thermal energy. Convection transfers energy away from the
observers as hot air rises, while conduction is negligibly slow here. Skin is very sensitive to infrared radiation, so
you can sense the presence of a fire without looking at it directly. (credit: Daniel O’Neil)
Source: Moses, Ntam, Calculus Based General Physics 2 (PHYS 311). OpenStax CNX. Dec
20, 2019. http://cnx.org/contents/9c45112c-cc2e-4e89-8f91-5e3c25cd65f2@1.140.

The energy of electromagnetic radiation varies over a wide range, depending


on the wavelength: A shorter wavelength (or higher frequency) corresponds to
a higher energy. Because more heat is radiated at higher temperatures, higher
temperatures produce more intensity at every wavelength but especially at
shorter wavelengths.

In visible light, wavelength determines color—red has the longest wavelength


and violet the shortest—so a temperature change is accompanied by a color
change. For example, an electric heating element on a stove glow from red to
orange, while the higher-temperature steel in a blast furnace glows from yellow
to white. Infrared radiation is the predominant form radiated by objects cooler
than the electric element and the steel. The radiated energy as a function of
wavelength depends on its intensity, which is represented in Figure 11.8 by the
height of the distribution.
3 | Physics for Engineers 172

Figure 11.8: (a) A graph of the spectrum of electromagnetic waves emitted from an ideal radiator at three
different temperatures. The intensity or rate of radiation emission increases dramatically with temperature, and the
spectrum shifts down in wavelength toward the visible and ultraviolet parts of the spectrum. The shaded portion
denotes the visible part of the spectrum. It is apparent that the shift toward the ultraviolet with temperature makes
the visible appearance shift from red to white to blue as temperature increases. (b) Note the variations in color
corresponding to variations in flame temperature.
Source: Moses, Ntam, Calculus Based General Physics 2 (PHYS 311). OpenStax CNX. Dec
20, 2019. http://cnx.org/contents/9c45112c-cc2e-4e89-8f91-5e3c25cd65f2@1.140.

The rate of heat transfer by radiation also depends on the object’s color. Black
is the most effective, and white is the least effective. On a clear summer day,
black asphalt in a parking lot is hotter than adjacent gray sidewalk, because
black absorbs better than gray (Figure 11.9).

Figure 11.9: The darker pavement is hotter than the lighter pavement (much more of the ice on the right has
melted), although both have been in the sunlight for the same time. The thermal conductivities of the pavements
are the same.
Source: Moses, Ntam, Calculus Based General Physics 2 (PHYS 311). OpenStax CNX. Dec
20, 2019. http://cnx.org/contents/9c45112c-cc2e-4e89-8f91-5e3c25cd65f2@1.140.
3 | Physics for Engineers 173

The reverse is also true — black radiates better than gray. Thus, on a clear
summer night, the asphalt is colder than the gray sidewalk, because black
radiates the energy more rapidly than gray. A perfectly black object would be
an ideal radiator and an ideal absorber, as it would capture all the radiation
that falls on it.

Figure 10: A black object is a good absorber and a good radiator, whereas a white, clear, or silver object is a
poor absorber and a poor radiator.
Source: Moses, Ntam, Calculus Based General Physics 2 (PHYS 311). OpenStax CNX. Dec
20, 2019. http://cnx.org/contents/9c45112c-cc2e-4e89-8f91-5e3c25cd65f2@1.140.

In contrast, a perfectly white object or a perfect mirror would reflect all


radiation, and a perfectly transparent object would transmit it all (Figure 10).
Such objects would not emit any radiation. Mathematically, the color is
represented by the emissivity e. A “blackbody” radiator would have an e = 1,
whereas a perfect reflector or transmitter would have e = 0. For real examples,
tungsten light bulb filaments have an e of about 0.5, and carbon black (a
material used in printer toner) has an emissivity of about 0.95.

To see that, consider a silver object and a black object that can exchange heat by
radiation and are in thermal equilibrium.We know from experience that they
will stay in equilibrium (the result of a principle that will be discussed at length
in Second Law of Thermodynamics).

For the black object’s temperature to stay constant, it must emit as much
radiation as it absorbs, so it must be as good at radiating as absorbing. Similar
considerations show that the silver object must radiate as little as it absorbs.
Thus, one property, emissivity, controls both radiation and absorption.

Finally, the radiated heat is proportional to the object’s surface area, since every
part of the surface radiates. If you knock apart the coals of a fire, the radiation
increases noticeably due to an increase in radiating surface area.
3 | Physics for Engineers 174

The rate of heat transfer by emitted radiation is described by the Stefan-


Boltzmann Law of Radiation:

𝑷 = 𝝈𝑨𝒆𝑻𝟒 𝑬𝒒. 𝟏𝟏. 𝟑


Where,

𝜎 = 5.67 × 10−8 𝐽/𝑠 · 𝑚2 · 𝐾 4 → Stefan-Boltzmann constant, a


combination of fundamental constants of nature

A → surface area of the object

e → emissivity

T → temperature in kelvins

The proportionality to the fourth power of the absolute temperature is a


remarkably strong temperature dependence. It allows the detection of even
small temperature variations.

Images called thermographs can be used medically to detect regions


ofabnormally high temperature in the body,perhaps indicative ofdisease.
Similar techniques can be used to detect heatleaksinhomes (Figure 11.11),
optimize performance of blast furnaces, improve comfort levels in work
environments, and even remotely map Earth’s temperature profile.

Figure 11.11: A thermograph of part of a building shows temperature variation, indicating where heat transfer
to the outside is most severe. Windows are a major region of heat transfer to the outside of homes. (credit: US
Army)
Source: Moses, Ntam, Calculus Based General Physics 2 (PHYS 311). OpenStax CNX. Dec
20, 2019. http://cnx.org/contents/9c45112c-cc2e-4e89-8f91-5e3c25cd65f2@1.140.

The Stefan-Boltzmann equation needs only slight refinement to deal with a


simple case of an object’s absorption of radiation from its surroundings.
Assuming that an object with a temperature T1 is surrounded by an
environment with uniform temperature T2, the net rate of heat transfer by
radiation is:
3 | Physics for Engineers 175

𝑷𝑛𝑒𝑡 = 𝝈𝑨𝒆(𝑻𝟐 𝟒 − 𝑻𝟏 𝟒 ) 𝑬𝒒. 𝟏𝟏. 𝟒

where e is the emissivity of the object alone. In other words, it does not matter whether
the surroundings are white, gray, or black: The balance of radiation into and out of the
object depends on how well it emits and absorbs radiation. When T 2 >T1, the quantity
Pnet is positive, that is, the net heat transfer is from hot to cold.

Before doing an example, we have a complication to discuss: different


emissivities at different wavelengths. If the fraction of incident radiation an
object reflects is the same at all visible wavelengths, the object is gray; if the
fraction depends on the wavelength, the object has some other color.

For instance, a red or reddish object reflects red light more strongly than other
visible wavelengths. Because it absorbs less red, it radiates less red when hot.
Differential reflection and absorption of wavelengths outside the visible range
have no effect on what we see, but they may have physically important effects.
Skin is a very good absorber and emitter of infrared radiation, having an
emissivity of 0.97 in the infrared spectrum. Thus, in spite of the obvious
variations in skin color, we are all nearly black in the infrared. This high
infrared emissivity is why we can so easily feel radiation on our skin. It is also
the basis for the effectiveness of night-vision scopes used by law enforcement
and the military to detect human beings.

Example: Radiation Calculation______________________________________


7. What is the rate of heat transfer by radiation of an unclothed person
standing in a dark room whose ambient temperature is 22.0°C? The person
has a normal skin temperature of 33.0°C and a surface area of 1.50 m2. The
emissivity of skin is 0.97 in the infrared, the part of the spectrum where the
radiation takes place.

Given: T2 = 22.0°C = 295 K


T1 = 33.0°C = 306 K
A = 1.50 m2
e = 0.97
Required: P
Solution:
We can solve this by using the equation for the rate of radiative heat transfer.

𝑃𝑛𝑒𝑡 = 𝜎𝐴𝑒(𝑇2 4 − 𝑇14 )


𝐽
= (5.67 × 10−8 𝑠 · 𝑚2 · 𝐾 4 ) (1.50 m2 )(0.97)((295 𝐾)4 − (306 𝐾 )4 )
3 | Physics for Engineers 176

𝑱
= −𝟗𝟗 𝒔 = −𝟗𝟗 𝑾

This value is a significant rate of heat transfer to the environment (note the minus
sign), considering that a person at rest may produce energy at the rate of 125W
and that conduction and convection are also transferring energy to the
environment. Indeed, we would probably expect this person to feel cold. Clothing
significantly reduces heat transfer to the environment by all mechanisms because
clothing slows down both conduction and convection and has a lower emissivity
(especially if it is light-colored) than skin.

The average temperature of Earth is the subject of much current discussion.


Earth is in radiative contact with both the Sun and dark space, so we cannot use
the equation for an environment at a uniform temperature. Earth receives
almost all its energy from radiation of the Sun and reflects some of it back into
outer space.

Conversely, dark space is very cold, about 3 K, so that Earth radiates energy
into the dark sky. The rate of heat transfer from soil and grasses can be so rapid
that frost may occur on clear summer evenings, even in warm latitudes. The
average temperature of Earth is determined by its energy balance. To a first
approximation, it is the temperature at which Earth radiates heat to space as
fast as it receives energy from the Sun.

An important parameter in calculating the temperature of Earth is its emissivity


(e). On average, it is about 0.65, but calculation of this value is complicated by
the great day-to-day variation in the highly reflective cloud coverage. Because
clouds have lower emissivity than either oceans or land masses, they reflect
some of the radiation back to the surface, greatly reducing heat transfer into
dark space, just as they greatly reduce heat transfer into the atmosphere during
the day. There is negative feedback (in which a change produces an effect that
opposes that change) between clouds and heat transfer; higher temperatures
evaporate more water to form more clouds, which reflect more radiation back
into space, reducing the temperature.

The often-mentioned greenhouse effect is directly related to the variation of


Earth’s emissivity with wavelength (Figure 11.12). The greenhouse effect is a
natural phenomenon responsible for providing temperatures suitable for life
on Earth and for making Venus unsuitable for human life. Most of the infrared
radiation emitted from Earth is absorbed by carbon dioxide (CO2) and water
(H2O) in the atmosphere and then re-radiated into outer space or back to Earth.
Re-radiation back to Earth maintains its surface temperature about 40°C higher
than it would be if there were no atmosphere. (The glass walls and roof of a
greenhouse increase the temperature inside by blocking convective heat losses,
not radiative losses.)
3 | Physics for Engineers 177

Figure 11.12: The greenhouse effect is the name given to the increase of Earth’s temperature due to
absorption of radiation in the atmosphere. The atmosphere is transparent to incoming visible radiation and most of
the Sun’s infrared. The Earth absorbs that energy and re-emits it. Since Earth’s temperature is much lower than
the Sun’s, it re-emits the energy at much longer wavelengths, in the infrared. The atmosphere absorbs much of
that infrared radiation and radiates about half of the energy back down, keeping Earth warmer than it would
otherwise be. The amount of trapping depends on concentrations of trace gases such as carbon dioxide, and an
increase in the concentration of these gases increases Earth’s surface temperature.

Source: Moses, Ntam, Calculus Based General Physics 2 (PHYS 311). OpenStax CNX. Dec
20, 2019. http://cnx.org/contents/9c45112c-cc2e-4e89-8f91-5e3c25cd65f2@1.140.

The greenhouse effect is central to the discussion of global warming due to


emission of carbon dioxide and methane (and other greenhouse gases) into
Earth’s atmosphere from industry, transportation, and farming. Changes in
global climate could lead to more intense storms, precipitation changes
(affecting agriculture), reduction in rain forest biodiversity, and rising sea
levels.
3 | Physics for Engineers 178

Assessment
Answer the two assessments below (Exercise and Quiz). Write/Encode the
assessments in a long bond paper.

You will be graded based on the following rubrics:


RUBRIC

A. Essay/Quiz

Criteria 5 4 3 2 1
The student was The student
able to elicit ideas The student is unable to
and correct was not only elicit ideas
The student
able to elicit and concepts
understanding was able to The student
ideas and from the
from the reading, elicit ideas and was able to
Understanding correct reading
shows evidence of correct elicit ideas
40% understanding indicating
internalizing these, understanding from the
but also shows that he/she
and contributed from the reading
evidence of has not read
additional thoughts reading.
internalizing the
to the core idea. these. prescribed
reading.
Shows correct Shows
Shows correct
content with Shows irrelevant and
content with
Content additional content Shows correct partial erroneous
evidence
40% resources not content. erroneous content. Did
sourced from
limited to the content. not follow
the module.
module. directions.
Submitted Submitted
Submitted
Submitted on or Submitted after after after
Timeliness after the
before the the deadline: deadline: deadline:
20% deadline:
deadline. 1 day – 3 days 9 days – 15 16 days - 30
4 days – 8 days
days days.
Note for timeliness: 0 points for late submission more than 30 days from the deadline.

Exercise No. 11
1. How is heat transfer related to temperature?
2. When our bodies get too warm, they respond by sweating and increasing blood circulation to the
surface to transfer thermal energy away from the core. What effect will those processes have on
a person in a 40.0-°C hot tub?
3. Some electric stoves have a flat ceramic surface with heating elements hidden beneath. A pot
placed over a heating element will be heated, while the surface only a few centimeters away is
safe to touch. Why is ceramic, with a conductivity less than that of a metal but greater than that
of a good insulator, an ideal choice for the stove top?
4. Why are thermometers that are used in weather stations shielded from the sunshine? What does
a thermometer measure if it is shielded from the sunshine? What does it measure if it is not?
5. Broiling is a method of cooking by radiation, which produces somewhat different results from
cooking by conduction or convection. A gas flame or electric heating element produces a very
3 | Physics for Engineers 179

high temperature close to the food and above it. Why is radiation the dominant heat-transfer
method in this situation?

Quiz No. 11

1. (a) Calculate the rate of heat conduction through house walls that are 13.0 cm thick and have an
average thermal conductivity twice that of glass wool. Assume there are no windows or doors.
The walls’ surface area is 120m2 and their inside surface is at 18.0°C, while their outside surface
is at 5.00°C. (b) How many 1-kW room heaters would be needed to balance the heat transfer
due to conduction?
2. Suppose you stand with one foot on ceramic flooring and one foot on a wool carpet, making
contact over an area of 80.0 cm2 with each foot. Both the ceramic and the carpet are 2.00 cm
thick and are 10.0°C on their bottom sides. At what rate must heat transfer occur from each foot
to keep the top of the ceramic and carpet at 33.0°C?
3. A man consumes 3000 kcal of food in one day, converting most of it to thermal energy to maintain
body temperature. If he loses half this energy by evaporating water (through breathing and
sweating), how many kilograms of water evaporate?
4. At what net rate does heat radiate from a 275 m2 black roof on a night when the roof’s
temperature is 30°C and the surrounding is 15°C? The emissivity of the roof is 0.9.
5. Radiation makes it impossible to stand close to a hot lava flow. Calculate the rate of heat transfer
by radiation from 1.00 m2 of 1200°C of fresh lava into 30.0°C surrounding assuming lava’s
emissivity is 1.00.

11.3. References
Padua, A.L. & Crisostomo, R. M. (2010). Science and Technology IV Practical and
Explorational Physics Modular Approach, 2nd Edition, Philippines: Vibal
Publishing House Inc.

Young, H.D. (1992). University Physics, 8th Edition, Addison-Wesley Pub. Co.
Review of Simple Harmonic Motion. Flipping Physics. March 23, 2020.
https://www.flippingphysics.com/ap1-shm-review.html.
Collection: OpenStax University Physics - All Volumes Edited by: Cengage
WebAssign URL: https://legacy.cnx.org/content/col12105/1.1/ Copyright:
Cengage WebAssign License: http://creativecommons.org/licenses/by/4.0/
Based on: University Physics <http://legacy.cnx.org/content/col11994/1.1>
arranged by OpenStax University Physics.

Del Rosario, A.C.M. (2004). College Physics.Philippines

Moses, Ntam, Calculus Based General Physics 2 (PHYS 311). OpenStax CNX.
Dec 20, 2019. http://cnx.org/contents/9c45112c-cc2e-4e89-8f91-
5e3c25cd65f2@1.140.
3 | Physics for Engineers 180

Open-Stax College. College Physics. http://cnx.org/contents/031da8d3-


b525-429c-80cf-6c8ed997733a/College_Physics.

11.4. Acknowledgement

The images, tables, figures, and information contained in this module were
taken from the references cited above.
3 | Physics for Engineers 181

Appendix A
6700

You might also like